Sie sind auf Seite 1von 35

1 From patterns to generalizations:

sequences, series and proof


Concepts
Patterns
How can a finite area be enclosed by an
infinite perimeter?
The diagrams here are the first four iterations to
create a Koch snowflake.
Describe what you observe about:

T

One does not have to look far and wide to find
■ Generalizations • the perimeter as you move from the equilateral
visual patterns, but some questions immediately triangle through to the fourth iteration
arise. Microconcepts • the area enclosed as you move from the triangle to
■ Arithmetic and geometric sequences and the fourth iteration.
series

F
What changes would you expect in the fifth iteration?
Introduction to limits
Developing inquiry

■ Sum of series How would you physically measure the perimeter at
the fifth iteration if the original triangle had sides of
Permutations and combinations
skills

1m in length?
■ Proof
Binomial theorem What happens if you start with a square instead of an
■ To come

A
equilateral triangle?
Can a finite volume enclose an infinite area?

Before you start


Click here for help

R
You should know how to: Skills check with this skills check

1 Solve linear algebraic equations. 1 Solve the following equations:


eg: x - 3(x + 5) = 20 - 3x a 3x + 5(x - 4) = 20x + 4
⇒ x - 3x - 15 = 20 - 3x x 1 x 3
b 
Can these patterns be explained ⇒ −2x - 15 = 20 - 3x

D
2x  1 2 x  1
mathematically? ⇒ x = 35
2 Simplify surds. 2 Simplify the following:
Can patterns in numbers be
useful in real-life situations? eg simplify
2 a 1 2
1 2 1 2
2 2
2
 

2 1 2
22 
2  2
b
1 3
1 2 
1 2 1 2 1 2  
3 Manipulate algebraic fractions. 3 Simplify:
x +3 2 3x x 1 2
eg simplify = −  
x x +1 x −1 x  1 2x  1 x  1
(x  3)(x  1)(x  1)  2x(x  1)  3x 2(x  1)

x(x  1)(x  1)
(x  3)(x 2  1)  2x 2  2x  3x 3  3x 2

x(x 2  1)
What information would you require to x 3  x  3x 2  3  2x 2  2x  3x 3  3x 2

choose the best loan offer? What other If you take out a loan to buy a x(x 2  1)
scenarios could this be applied to? car how can you determine the 
2x 3  2x 2  3x  3
actual amount it will cost? x(x 2  1)

2 3
1 FROM PATTERNS TO GENERALIZATIONS: SEQUENCES, SERIES AND PROOF 1.1
1.1 Sequences, series and sigma Look at the following sequences of numbers and identify the rule

Number and algebra


which would help you obtain the next term.
notation i 7, 5, 3, 1, …
ii 2, 4, 8, 16, …
Opening investigations iii 1, 3, 9, 27, …
You are going to start this chapter by doing some simple arithmetic

T
with the aim of recognizing patterns. The challenge is for you to Sequences may be finite or infinite.
understand and explain the patterns that emerge. In Investigation 2,
you will be asked to propose a conjecture, which is a rule generalizing The sequence 7, 5, 3, 1, -1, -3 is a finite sequence with six terms,
findings based on observed patterns. whereas the sequence 7, 5, 3, 1, -1, -3, … is an infinite sequence with
an infinite number of terms. The distinction is indicated by the ellipsis

F
(…) at the end of the sequence.
Investigation 1 International-
A sequence is sometimes written in terms of the general term as {ur},
Work out the following products:
mindedness where r can take values 1, 2, 3, … HINT
1 × 1            11 × 11          111 × 111            1111 × 1111 Where did numbers If the sequence is finite then r will terminate at some point. {ur} represents the
1 What pattern do you see emerging? come from? sequence whereas

A
The sequence {ur} = {3r − 1}, where r ∈ ℤ+ represents the infinite
2 Does this continue as you make the string of 1’s longer? 1
sequence 2, 5, 8, 11, …, whereas the sequence ur    2 , where ur represents the
r  r th term.
3 Can you predict when this pattern stops and explain why this happens? 1 1 1 1
r ∈ ℤ+, r ≤ 5 represents the finite sequence 1, , , , .
4 9 16 25
All the terms in a sequence added together are called a series. Like

R
sequences, series can be finite or infinite.
Investigation 2
The series obtained by adding the six terms of the sequence 7, 5, 3, 1,
This diagram represents the floor of a room covered with square
-1, -3 is 7 + 5 + 3 + 1 − 1 − 3 = 12. This is a finite series. The sum
tiles. It has a total of nine tiles along the main diagonals (shaded),
1 + 3 + 9 + 27 + 81 + … continues indefinitely and is an infinite series.
and five tiles on each side. 25 tiles are used to cover the floor
completely. The set of positive integers ℤ+ can be written as {1, 2, 3, 4, 5, …, r, …}

D
where the letter r is used to represent the general term. If the positive
Another room has a total of 13 square tiles along the diagonals.
integers which are multiples of 5 are considered, then the set {5, 10,
1 How many square tiles are there on each side in this other 15, 20, …5r, …} is obtained. In this case the general term is 5r where r
room? is any positive integer. The harmonic series is the infinite sum of the TOK
1 1 1
2 How many tiles are needed to completely cover the floor? reciprocals of positive integers, ie 1    ...   ... Do the names that we
2 3 r
3 What if the total number of tiles along the diagonals is 15? Series can be represented in compact form using sigma (∑) notation. give things impact how
This makes use of the general term written in terms of r, which often we understand them?
4 What if there is a total of 135 tiles along the diagonals? represents a positive integer.
5 What if the total number of squares along the diagonals is an even number? The sum of the first 10 positive integers can be written as follows using
6 Continue to generate data to help you form a conjecture. Can you explain why this rule holds true? sigma notation:

7 How can you write the generalization concisely? The largest value that r can take
10
8  Conceptual Why is an algebraic expression more useful than generating numerical values? r
r 1

The smallest value that r can take


A sequence is a list of numbers that is written in a defined order,
ascending or descending, following a specific rule. Each of the numbers Read this as “The sum of r, from r = 1 to r = 10.”
making up a sequence is called a term of that sequence. Sometimes a If you want to write the sum of the positive multiples of 5 less than
sequence is also referred to as a progression. 100, then you first need to think of the general term, which is 5r, and
then establish the range of values that r can take. The smallest positive

4 5
1 FROM PATTERNS TO GENERALIZATIONS: SEQUENCES, SERIES AND PROOF 1.1
multiple of 5 is 5 in which case r = 1, and since you want the largest Example 2 shows how to find the terms of a sequence represented by

Number and algebra


multiple of 5 to be 100 the largest value that r can take is 20 because its general term.
100 = 5 × 20.
20

 5r
5  10  15  ...  100 
r 1
Example 2
Sometimes you will also have to interpret a sum given in sigma Write down the first three terms of each of the following sequences:

T
notation and expand it into individual terms. For example: HINT (1)r 
a {ur} = {5r − 2}, r ∈ ℤ+ b ur   2  , r   

(2r + 1) = (2 × 0 + 1) + (2 × 1 + 1) + (2 × 2 + 1) + (2 × 3 + 1) + (2 × 4 + 1) = 1 + 3 + 5 + 7 + 9 In this case
4


r =0 the series
 r 

starts r = 0. a u1 = 5 × 1 − 2 = 3 Substitute values 1, 2 and 3 for r.


In Example 1 you will learn how to look for a pattern and write the
u2 = 5 × 2 − 2 = 8

F
general term.
u3 = 5 × 3 − 2 = 13
Example 1 3, 8, 13
For each of the following sequences, write the next three terms and find the general term: (1)1
1 2 3 4 b u1   1 Substitute values 1, 2 and 3 for r.

A
a 2, 7, 12, 17, …  b  2, 6, 12, 20, …  c  , , , , ...  d  5, 10, 20, 40, … 12
2 3 4 5
(1)2 1

u2 
a 
The next three terms of this sequence are Note that at each step we add 5 to get the 22 4
22, 27, 32. next term. (1)3 1
u3   
The sequence can be written as: Write the sequence using pattern noticed. 32 9
1 1
2, 2 + 5, 2 + 10, 2 + 15
 1, , 

R
4 9
= 2, 2 + (1 × 5), 2 + (2 × 5), 2 + (3 × 5), …,

2 + (r − 1) × 5
The general term is 2 + (r − 1) × 5 = 5r − 3,
 Example 3 shows how to represent a given sequence by its general
where r can take the values 1, 2, 3, … term after recognizing a pattern.
b The next three terms are 30, 42, 56. Note that the given terms can be written as:

D
The
 sequence can be written as 1 × 2, 2 × 3, 1 × 2, 2 × 3, 3 × 4, 4 × 5, … Example 3
3 × 4, 4 × 5, …, r × (r + 1), …
Write each of the following sequences using the general term:
The
 general term is r × (r + 1), where r 1 2 3 4
can take the values 1, 2, 3, … a 3, 6, 9, 12, … b 2, −10, 50, −250 c , , , , ...
3 5 7 9
5 6 7
c The next three terms are , ,  . The pattern here is easy to follow.
6 7 8 a 3, 6, 9, 12, … This is an infinite sequence of the positive
The sequence can be written as: multiples of 3.
1 2 3 4 r
{ur} = {3r}, r ∈ ℤ+
, , , , ..., , ...
2 3 4 5 r 1
The
 general term is
r
, where r can b 2, −10, 50, −250 This finite sequence can be written as:
r 1 {ur} = {2(−5)r−1},  r ∈ ℤ+, r ≤ 4 2, 2  (5), 2  25, 2  (125)
take the values 1,2,3, …
which can be rewritten in terms of
d 
The next three terms are 80, 160, 320. Each term is obtained by multiplying the powers of  5:
The
 general term is 5 × 2r−1, where r can previous term by 2.
 2  (5)0 , 2  (5)1 , 2  (5)2 , 2  (5)3
take the values 1, 2, 3, …
1 2 3 4
c
, , , , ... In this infinite sequence, the numerators
3 5 7 9
are the positive integers and the denomin-
ur  
r   ators are successive odd integers greater
HINT  , r  
  1
2r than 1.
You can check the answers by putting r = 5,6,7 in the general term obtained
in each case.

6 7
1 FROM PATTERNS TO GENERALIZATIONS: SEQUENCES, SERIES AND PROOF 1.1
Example 4 shows how to expand a series written in sigma notation.
Exercise 1A

Number and algebra


Example 4
5
r
1 For each of the following sequences, write the c  3r  1
For each of the following series written in sigma notation, write the first five terms: next three terms and find the general term: r 1
4
a 3,   4.5,   6,  7.5,  … 5

10 
r 1 d
 r(r  1)   1 c 
r
a b 2
r
r 1 r  1
2 r 1

T
r 1 r 1 b 17,   14,   11,    8,  … 3

a
10

 r(r  1)  1  0  2  1  3  2  4  3  5  4  ... Substitute r = 1 to 5 for the first c 3,   9,  27,   81,   … e r
r 0
2
3
r 1 through to the fifth term. 1 4 7 10
d , , , , ... 5 For each of the following series written in
   = 0 + 2 + 6 + 12 + 20 + … + 90 Simplify. 4 7 10 13
sigma notation, write the first five terms:
1 1 1 1

F
e , , , , ... 
r 1
 2 12 30 42 a 
  1
r
b r 2
r 1 r2
r 1 2 Write down the first five terms of each of

(1)r

= (−1)1 × + ×12 +(−1)2× 22 (−1)3 32 the following sequences: b  2r 2
1
+ (−1)4 × 42 + (−1)5 × 52 + … a ur = 3 − 2r r 1
20

 r(5r  1)

A
= −1 + 4 − 9 + 16 − 25 + … r c
b ur 
2r  1 r 1

c  r  1   1  1  2  1  3  1  4  1  5  1
 5

r 1 2r  1 2  1 4  1 6  1 8  1 10  1
c ur = 2r + (−1)r r d 2
r 0
r
3
4 5 6 d ur = (−1)r × 2
 2  1     ... ∞
5 7 9 3
e ur  r 1 e ∑r r

R
2 r =1
3 Write each of the following sequences using 6 Write each of the following series in sigma
In Example 5 you will see how a given series can be written in sigma the general term:
notation.
TOK notation:
a 5,   10,   15,   20,  … a 8 + 5 + 2 − 1 − 4
Is mathematics a language?
b 6,  14,  22,  30,  … b 3 + 10 + 21 + 36 + 55

D
1 1 1 1 1 1 3 2 5
c , , , , ... c 0       ...
2 4 8 16 3 2 5 3 7
Example 5 1 1 1
d 1 + 9 + 25 + 49 + 81
d 1,  , ,  , ...
e 3k + 6k + 9k + 12k + 15k
Write each of the following series in sigma notation: 3 9 27
a 3 + 11 + 19 + 27 + 35   b  1 − 1 + 1 − 1 + 1 − 1 + …  c  −6 + 12 − 24 + 48 − 96 + 192 e 0,   3,  8,   15,   …
a 3 + 11 + 19 + 27 + 35 This is a finite series which can be written as: 4 Write each of the following series in full:
4
3 + (3 + 8) + (3 + 16) + (3 + 24) + (3 + 32)
 2r(1  r)
5
  8r  5 a
= 3 + (3 + 1 × 8) + (3 + 2 × 8) + (3 + 3 × 8)
r 1
r 1
+ (3 + 4 × 8) 5
b (1) r
r 0
r 2

The general term is 3 + (r − 1) × 8 = 8r − 5.


b 1 − 1 + 1 − 1 + 1 − 1 + … This is an infinite series. Each term

oscillates between −1 and +1 and the
 (1)r 1
r 1
general term is (−1)r−1.
Developing inquiry skills
Now go back to the opening question. Suppose the length of each side
c −6 + 12 − 24 + 48 − 96 + 192 This is a finite series with oscillating signs of the first triangle is 81 cm. Can you work out the length of each side of
6
and each term is the next multiple of 6. the figure in each iteration? Tabulate your results and try to find a pattern
  (1) 6r r
and then make a conjecture.
r 1

8 9
1 FROM PATTERNS TO GENERALIZATIONS: SEQUENCES, SERIES AND PROOF 1.2
1.2 Arithmetic and geometric If the difference between two consecutive numbers in a sequence is constant

Number and algebra


then it is an arithmetic sequence or an arithmetic progression. The constant
sequences and series difference is called the common difference and is denoted by d.

Investigation 3 Consider how an arithmetic sequence with first term u1 and common
HINT

T
Whenever you go through airport security you have to place your hand luggage, coat, phone, etc into a tray difference d grows:
First term A recursive equation
that goes on a conveyer belt which then takes it through an x-ray scanner. u1
is one in which the
When answering the following questions, you can assume Second term u2 = u1 + d next term is defined
the following: Third term u3 = u2 + d = u1 + 2d as a function of

F
• Trays are placed on the conveyer belt with no gaps earlier terms. In the
between them. Fourth term u4 = u3 + d = u1 + 3d case of an arithmetic
• The length of each tray is 60 cm. This leads to the general term un = u1 + (n − 1)d. sequence the
recur­sive equation is
• The conveyer belt is moving at 10 cm per second.
un = u(n−1) + d.
• Each person uses three trays. An arithmetic sequence with first term u1 and common difference d has

A
general term un = u1 + (n − 1)d.
1 Copy and complete the following table:

Number of people ahead Distance of your first Waiting time, The next four examples show you how to use the general term
of you tray to machine, d (m) T (s) formula to answer different types of questions.
0 0 0
1 1.8 Example 6

R
2 36 The fourth term of an arithmetic sequence is 18 and the common difference is –5. Determine
. . . the first term and the nth term.
. . .
. . . u4 = u1 + 3 × (−5) = 18
. . . ⇒ u1 = 18 + 15 = 33 Using un = u1 + (n − 1)d.

D
n un = 33 + (n − 1) × (−5)
2 What patterns do you see emerging? ⇒ un = 38 − 5n
3 Now assume that there is a 30 cm gap separating trays belonging to different passengers. Construct and
complete a table similar to the one above.
4 How have the patterns changed? Example 7
5 What happens if the distance between the trays of individual passengers changes to 50 cm? 60 cm? 80 cm? Find the number of terms in the following arithmetic sequences:
6 How have the patterns changed? a 20, 23, 26, …, 83  b  34, 30, 26, …, −30  c  6a, 4a, 2a, …, −22a
7 Factual What do you notice about consecutive terms in the second and third columns? a u1 = 20,   d = 3 d = 23 − 20 = 3
8 Factual How would you generalize the relationship between the distance from the machine to your first un = 17 + 3n = 83 Using un = u1 + (n − 1)d.
tray and the number of people ahead of you?
⇒ n = 22 Solve the linear equation to obtain n.
9 Factual Write down the relationship between the waiting time and the number of people ahead of you.
b u1 = 34, d = −4
10  Conceptual How did the recognition of patterns help you make predictions about waiting times? un = 38 − 4n = −30
⇒ n = 17
Arithmetic sequences and series c u1 = 6a, d = −2a
A growth pattern that is represented by a linear relationship is also un = 8a − 2an = −22a
known as an arithmetic sequence, which is defined as follows: ⇒ n = 15

10 11
1 FROM PATTERNS TO GENERALIZATIONS: SEQUENCES, SERIES AND PROOF 1.2
Example 8 Michela and Grisha start by laying out the cards in ascending order. Michela International-

Number and algebra


takes away the first card and the last card and notes that their sum is 56. mindedness
Three numbers are consecutive terms of an arithmetic sequence. The sum of the three Grisha then takes the first and last card from the cards that remain and
numbers is 45, and their product is 3240. Find the three numbers. notes that their sum is also 56. They continue to do this until just one card Karl Friedrich Gauss
is left. (1777–1855) was
Let the three numbers be u − d, u, u + d a renowned German

T
3u = 45 Taking the sum of the numbers. 1 Which card will this be? mathematician. It is
⇒ u = 15 2 Using the information above, how would you determine the sum of the first said that when he was
55 positive integers? in primary school his
u(u2  d 2 ) 
3240 Take the product.
3 What if you wanted to find the sum of the first 1000 positive integers? teacher challenged
3240
 152  d
2
 216 Substitute u = 15 and divide by 15. 4  Factual Explain the importance of the actual number of terms him to find the sum

F
15 added. of the numbers from
 d 2  225  216  9 1 to 100. To the
5 Repeat the process for finding the sum of:
d3 The two values of d produce two possible teacher’s amazement,
a the first 100 even numbers
sequences: Gauss gave the
b the positive multiples of 3 less than 1000.
12, 15, 18 or correct answer

A
The three numbers are 12, 15 and 18. 18, 15, 12 Conceptual How was Michela’s and Grisha’s method more efficient?
6  almost immediately.
He came to the
answer by using
Reflect on Investigation 6 and explain how the method used is the method used in
equivalent to the direct derivation for the sum of an arithmetic series investigation 4.
Example 9 containing n terms, with first term u1 and common difference d as

R
The second term of an arithmetic sequence is 20 and the seventh term is 55. Find the first shown below.
term and the common difference of the sequence.
Sn = u1 + u1 + d + u1 + 2d + ... + u1 + (n − 2)d + u1 + (n − 1)d
u2 = u1 + d = 20 un = u1 + (n − 1)d
Sn = u1 + (n − 1)d + u1 + (n − 2)d + u1 + (n − 3)d + ... + u1 + d + u1
u7 = u1 + 6d = 55

D
2Sn = 2u1 + (n − 1)d + 2u1 + (n − 1)d + 2u1 + (n − 1)d + ... + 2u1 + (n − 1)d + 2u1 + (n − 1)d
⇒5d = 35 ⇒  d = 7
⇒ 2Sn = n [ 2u1 + (n − 1)d ]
u1 = 20 − 7 = 13 Solving simultaneously.
n
Or
⇒ Sn = [2u1 + (n − 1)d ]
2
u7 = u2 + 5d ⇒ 5d = 55 − 20 Write u7 in terms of u2.
This can be rewritten as follows:
⇒ 5d = 35 ⇒ d = 7 Solve for d. TOK
n
u1 = 20 − 7 = 13 S
n 2u1  (n  1)d  How is intuition used in
2 mathematics?
n
 u1  u1  (n  1)d 
2
The sum of an arithmetic sequence 
n
u1  un 
2

Investigation 4 The sum of a finite arithmetic series is given by


Miss Sandra, the grade 5 teacher, pairs up her students and gives each n n
pair 55 cards numbered from 1 to 55. She tells the students that she
Sn 2u1  (n  1)d  u1  un  where n is the number of terms in the
2 2
wants them to use these cards to find the sum of the numbers series, u1 is the first term, d is the common difference and un is the last term.
1 + 2 + 3 + … + 55.

12 13
1 FROM PATTERNS TO GENERALIZATIONS: SEQUENCES, SERIES AND PROOF 1.2
Example 10 Example 13

Number and algebra


The first term of an arithmetic series is 5 and the last term is –51. The series has 15 terms. The sum of an arithmetic series is given by Sn = n(2n − 3). Find the common difference and
Find: the first three terms of the series.
a the common difference S1 = u1 = −1 Using Sn = n(2n − 3).
b the sum of the series.

T
S2 = u1 + (u1 + d) ⇒ −2 + d = 2 Using S2 = u1 + u2.
a −51 = 5 + 14d Using un = u1 + (n − 1)d.
d=4
56
d  4 u1 = −1
14

F
n u2 = 3
b S  15 5  (51) 345 Using
Sn u1  un .
2
15
2 u3 = 7

Exercise 1B

A
Example 11
The first term of an arithmetic series is –7 and the fourth term is 23. The sum of the series is 1 Find the nth term of each of these 5 The sixth term of an arithmetic sequence
689. Find the number of terms in the series. sequences: is 37 and the common difference is 7. Find
a 3, 8, 13, 18, … the first term and the nth term.
u1 = −7 Using u4 = u1 + 3d. b 101, 97, 93, 89, … 6 The fifth term of an arithmetic sequence

R
23  7 is 0 and the 15th term is 180. Find the
u1  3d  23  d   10 c a − 3, a + 1, a + 5, a + 9, …
3 common difference and the first term.
d -20, -5, 10, 25, …
n
Sn = 689 = [ −14 + (n − 1) × 10] 2 Find the terms indicated in each of these
7 The sum of three consecutive terms of an
2 arithmetic sequence is 24 and their product
arithmetic sequences:
⇒ 10n − 24n − 1378 = 0
2
Rearrange and solve for n. is -640. Find the three numbers.
a 5, 11, 17, 23, … 15th term

D
⇒ 5n − 12n − 689 = 0
2 8 Jung Ho earned €38,000 when he started his
b 10, 3, -4, -11, … 11th term
⇒ (5n + 53)(n − 13) = 0 first job in the year 2000. He received a raise
c a, a + 2, a + 4, a + 6, … 17th term of €500 each consecutive year. Determine
⇒ n = 13, since n ∈  + d 16, 12, 8, 4, … (n + 1)th term how much he earned in 2017? Evaluate in
which year he would earn 50% more than
3 Find the number of terms in each of these
his original salary?
arithmetic sequences:
Why can n not be a rational or a negative number? a 16, 11, 6, …, -64 9 Find the value of each of the following
series:
b -108, -101, -94, …, 60
a 3 − 3 − 9 − 15 − 21 − … − 93
c -15, -19, -23, …, -95
Example 12 b 31 + 40 + 49 + … + 517
d 2a + 5, 2a + 3, 2a + 1, …, 2a − 23
28 c (a − 1) + (a + 2) + (a + 5) +   … + (a + 146)
Find the value of  5r  4 . 4 Determine the first term and the common
r 1
difference of the arithmetic sequences that 10 Find the value of each of the following
u1 = 1 Substitute r = 1 and r = 28 to find the first are generated by each of the following nth sums:
50 100
terms:
u28 = 140 − 4 = 136 and last terms. a  3r  8
b  7  8r
28 Using the formula
Sn
n
u1  un . a un = 5n − 7 r 1 r 1

S28 = (1 + 136) = 1918 2 b un = 3n + 11 20


2
c un = 6 − 11n
c  2ar  1, where a is a constant
r 1

d un = 2a + 2n + 1

14 15
1 FROM PATTERNS TO GENERALIZATIONS: SEQUENCES, SERIES AND PROOF 1.2
11 Find the sums of the following sequences 13 The third term of an arithmetic sequence Geometric sequences and series
is -8, and the sum of the first 10 terms of

Number and algebra


up to the term indicated:
In Investigation 5 you should have noticed that when filling out
a 4, -1, -6, … 15th term the sequence is -230. Find:
the table you would need to multiply the numbers in each row by a
b 3, 11, 19, … 10th term a the first term of the sequence particular constant to obtain the following column. In other words,
c 1,-4, -9, … 20th term b the sum of the first 13 terms. the ratio of a particular term to the previous term is a constant. Such
sequences are known as geometric sequences.
14 The sum of an arithmetic series is given by

T
12 Calculate the sum of an arithmetic series
with 25 terms given that the fifth term is 19 Sn = 6n − 3n2. Find the common difference HINT
and 10th term is 39. and the first four terms of the series. If the ratio of two consecutive terms in a sequence is constant then it is a The recursive equation
15 Calculate the sum of all the odd numbers geometric sequence or a geometric progression. We call the constant ratio for a geometric
the common ratio and denote it by r.
less than 300. sequence is

F
un = un−1 × r
Consider how a geometric sequence with first term u1 and common
ratio r grows:
Investigation 5 TOK
First term u1
The diagram below shows the first two iterations when constructing Sierpinski’s Is all knowledge

A
concerned with Second term u2 = u1r
triangle, named after the Polish mathematician Waclaw Sierpinski who first
described it in 1915. identification and Third term u3 = u2r = u1r2
use of patterns?
Fourth term u4 = ur = u1r3
This leads to the general term un = u1r n−1.

R
Stage 0 Stage 1 Stage 2 A geometric sequence with first term u1 and common ratio r has general term
un = u1r n−1, r ≠ 1, 0, −1, u1 ≠ 0.
1 Construct the next iteration (Stage 3).
2 Copy and fill out the table below by following these instructions:
Curiosities in geometric patterns

D
• Count the number of green triangles at each stage.
• If the sides of the triangle in stage 0 are each 1 unit long, what are • What happens if you have a sequence with first term u1 and
common ratio 1?
the lengths of sides of green triangles at each of the following three
stages? (Express your answers as rational numbers.) • What if the common ratio is 0?
• Now assume that the area of the triangle at Stage 0 is 1 unit². What • And what happens if the common ratio is -1?
is the area of each green triangle at each of the next three stages? In the first case, the sequence is just made up of constant terms u1. This
(Leave answers in fractional form.) is called a uniform sequence.
Stage 0 1 2 3 The next case is a sequence with first term u1 and all the other terms
Number of green triangles 1 are 0, which is a rather uninteresting sequence.
Length of one side of one The third case leads to what is known as an oscillating sequence:
1
green triangle u1, −u1, u1, −u1, …
Area of each green triangle 1
This oscillating sequence becomes particularly interesting if u1 = 1,
3 Factual What patterns emerge from each of the three rows of the table? which then leads to the sequence 1, -1, 1, -1, 1, …

4 Factual What do these three patterns have in common? If we try to take the sum of this series we run into some curious and
interesting results.
5 Based on your results, form a conjecture to obtain the numbers if you were
to extend the table further to stages 4, 5, 6, etc. We want to look at the sum S = 1 − 1 + 1 − 1 + 1 − 1 + …

6  Conceptual What has the mathematical model you have created


enabled you to do?

16 17
1 FROM PATTERNS TO GENERALIZATIONS: SEQUENCES, SERIES AND PROOF 1.2
There are various ways of looking at this sum. Possibly the most International-
Example 15

Number and algebra


intuitive way of finding this sum is by grouping the terms into pairs as
follows:
mindedness
Find the number of terms in each of these geometric sequences:
S = (1 − 1) + (1 − 1) + (1 − 1) + (1 − 1) + … = 0 + 0 + 0 + 0 + … = 0 The series S = 1 − 1
+1−1+1−1… a 0.15, 0.45, 1.35, …, 12.15 b 440, 110, 27.5, …, 0.4296875
But what happens if we pair the terms starting from the second term is known as Grandi’s 0.45 u2
a  
u1 0.15, r  3 Determine the value of r by computing .

T
instead of the first? series, after the Italian u1
0.15
S = 1 + (−1 + 1) + (−1 + 1) + (−1 + 1) + (−1 + 1) + …
mathematician Guido un = 0.15 × 3n−1 = 12.15 Use un = u1rn−1 to find n.
 =1+0+0+0+0…=1 Grandi (1671–1742). ⇒ 3n−1 = 81 = 34
You may want to
Yet another result is obtained if we look at the series from a different ⇒n=5
look into the history

F
perspective: and research on This sequence has 5 terms.
this sum by various 110
S  1  (1  1  1  1  1  1  ...) b  
u1 440, r  0.25 Use technology to find the value of n.
 1 S mathematicians after 440
its first appearance un = 440 × 0.25n−1 = 0.4296875 x 440 × (0.25)x
 2S 1
in Grandi’s book ⇒n−1=5 1 110

A
1 published in 1703.
S ⇒n=6 2 27.5
2
This sequence has 6 terms. 3 6.875
Why does this paradox arise and which is the correct answer? We
4 1.71875
have once more stumbled on the concept of infinity. If the number of
terms were to be made finite, then the result would be 0 if there are an 5 0.4296875
even number of terms, and 1 if the number of terms were odd, but an 6 0.1074219

R
infinite sum never ends. 7 0.0268555
The next examples show how to use the general term formula for a
geometric sequence to answer different types of questions.

Example 16

D
Example 14 The first term of a geometric sequence is 4 and the common ratio is –2. Determine which
term has the value of –2048?
Find the common ratio and write the next two terms of each
sequence: 4 × (−2)n−1 = −2048 Use technology to find the value of n.
a 2.5, 5, 10, … b 9, 3, 1, … c x, 2x3, 4x5, … ⇒ n = 10 x 4 × (− 2)x−1
5 u2 1 4
a 
r  2 Find r by calculating .
2.5 u1 2 -8
The next two terms are 20, 40. Use the recursive 3 16
equation to find the next
4 -32
b r 3 1 two terms.
9 3 5 64
1 1 HINT
The next two terms are , . 6 -128
3 9 This time the
7 256 formula uses
3
c 
2x
r  2x 2 8 -512 (x - 1) in the
x 9 1024 exponent so the
The next two terms are 8x7, 16x9. answer is n = 10
10 -2048

18 19
1 FROM PATTERNS TO GENERALIZATIONS: SEQUENCES, SERIES AND PROOF 1.2
Max then tried to generalize this result for a finite geometric series
Example 17 HINT

Number and algebra


with common ratio r and having n terms as follows:
The fourth term of a geometric sequence is 54 and the sixth term is 486. Determine the This formula can also

Sn = u1 + u1r + u1r 2 + u1r 3 + ... + u1r n −1
possible values of the common ratio. n −1 n 
⇒ (1 − r)Sn = u1 − u1r n be written as follows:
rSn = u1r + u1r + u1r + ... + u1r + u1r 
2 3
n
u1(r  1)
Use un = u1r n−1. n Sn , r  1.
u4  u1  r  54 
3
486 u1(1 − r ) r 1
⇒ Sn = , r ≠1

T
r   9
2

u6  u1  r  486
5
54 Divide the two expressions to obtain r2. 1− r This makes calculations
r  3 easier when r > 1.
Or The sum of a finite geometric series is given by
u u4  r 2 n
u1(1  r )

F
6
Sn , r 1
486 1 r
r  2
 9 where n is the number of terms, u1 is the first term and r is the common ratio.
54
 r 3

Investigation 6

A
Example 18 In the diagram, AB represents a piece of string A B
1 which is 100 cm long.
The first term of a geometric sequence is 16 and the common ratio is .
2 The string is cut in half and one of the halves, C D
1
Find the biggest term that is smaller than . CD, is placed underneath. The remaining half
1000 C D E

R
is now cut in half and one of the halves, DE, is
1
n 1
Use technology to find the value of n. placed next to CD. The process is continued as C D E F
16     0.001 shown in the diagram.
 2 x 16 × (0.5)(x−1) C D E F G
n 15 1 16
1 Copy and complete the table below.
 1
15
2 8
u15 = 16 ×  

D
 2 3 4 Line segment Length of string segment (cm) Total length of segments (cm)
4 2 CD 50 50
. . DE 25 75
Alternatively, you can use your GDC. . . EF
13 0.00390625 FG
14 0.001953125 2  Factual As this process continues indefinitely, what do you notice about the length of the line
15 0.000976563 segments? What about the total length of segments?
16 0.000488281 Modelling this scenario mathematically:
CD = 50 cm
1
The sum of a geometric sequence DE = 50 cm ×
2
= 25 cm
When trying to find the value of the series S = 1 + 3 + 9 + 27 + 81 + 243, 2
1  1
Max notices that this is a geometric series with common ratio 3, and EF = DE × = 50 cm ×   = 12.5 cm
that if he were to multiply the series by 3, he could more easily calculate 2  2
the sum as follows: 2 3
 1  1  1
3S  3  9  27  81  243  729  CD + DE + EF + FG = 50 + 50 ×   + 50 ×   + 50 ×  
 2  2  2
  3S  S  2S  729  1 728
S  1  3  9  27  81  243 
S  364
Continued on next page

20 21
1 FROM PATTERNS TO GENERALIZATIONS: SEQUENCES, SERIES AND PROOF 1.2
After four cuts have been made the sum of the length of string segments placed next to each other is a Use technology to copy and complete the following table:

Number and algebra


  1 n 
50 1     n 3n (−2)n (1.5)n (0.5)n (−0.2)n (−0.75)n
 2 
geometric sequence with four terms. Show that if n cuts are made this sum becomes     . 1 3 -2 1.5 0.5 -0.2 -0.75
1
1   2            
 2

T
Enter this into a table as shown below to see what happens as n gets bigger. 3            
4            
50  (1  (0.5)n )
5            
n (1  0.5)
6            
1 50

F
7            
2 75 8            
3 87.5 9            
4   10            
5  

A
6   1 Extend the table for different values of rn and larger values of n.
7   2  Conceptual What role does the value of the common ratio play in a
geometric series?
8  
3 Use your results to justify the following statements:
3 What would happen if you repeated this experiment, but this time you cut CD to be 2 of AB and DE to
3 a r > 1 ⇒ rn increases as n gets larger.
be 2 of the remaining piece of string?

R
3 b 0 < r < 1 ⇒ rn decreases as n gets larger.
4 Repeat the process using CD to be 3 of AB and DE to be 3 of the remaining piece of string. What if the c r < −1 ⇒ rn has a large absolute value, but its sign oscillates.
4 4
fraction used was 4 ? d −1 < r < 0 ⇒ rn has a small absolute value but its sign oscillates.
5 e When the value of r is close to (but still less than) 1, the value of rn
5 Write a short reflection on your results which includes answers to the following questions:
decreases more slowly but still gets close to zero when n gets larger.

D
• Factual Why were you asked to change the length of the string cut? u1
f If 1  r  1  Sn  as the value of n gets larger.
• Conceptual How has this process helped you analyse the situation? 1 r
• Conceptual What conclusion could you draw from your analysis?
n
u1(1  r )
The sum of n terms of a geometric
series is Sn , r  1.
Convergent and divergent series 1 r
An infinite geometric series is convergent when the sum tends to a When −1 < r < 1, rn approaches zero for very large values of n. The series
u
finite value as the number of terms gets bigger. If a geometric series therefore converges to a finite sum given by S  1 .
does not converge it is said to be divergent. 1 r

In Investigation 6, the series always converged to 100 cm, the length of


the original piece of string. The next examples demonstrate how to use the formulae for sums of
finite and infinite geometric series.

Investigation 7
Example 19
In Investigation 6, you would have noticed that you had a geometric series
in each case. You will now investigate a general geometric series in order to A geometric series has first term 3 and common ratio 2. Find the sum of the first five terms.
understand which conditions will make the series converge.
n
u1(1  r ) 3(1  25) 3(1  32) u1(1  r n )

For a geometric series, we know that Sn , r  1. S5   93 Use the formula Sn  .
1 r 1 2 1 1 r

22 23
1 FROM PATTERNS TO GENERALIZATIONS: SEQUENCES, SERIES AND PROOF 1.2
Example 20 Example 22

Number and algebra


i
1
7
Calculate the geometric series given by  2    . The sum of the first n terms of a geometric sequence is given by Sn = 7n − 1. Find the first
i 1  2 term and the common ratio of the sequence.
1 Enter i = 1 and i = 2 to find u1 and u2  and

u1 1,
r S1 = 7 − 1 = 6 ⇒ u1 = 6
hence r.

T
2
S2 = 49 − 1 = 48 ⇒ 6 + u2 = 48 ⇒ u2 = 42 Use formula given to find S1 and S2.
  1 7 
11 −    11 − 1  
r
42
 7
  2   128  u (1  r n )
S7 = =  1.98 Use the formula Sn  1 . 6
1 1 1 r
1−
2 2 The sum can be found using technology.

F
Or
7
1
i
Example 23
Using a GDC:  2     1.98 n
i 1  2 Determine how many terms are required for the sum of the geometric series given by  3  2i
to exceed 1000. i 1

A
u2
u
1 6, 
r  2
u1
n
6(1  2 ) u1(1  r n )
 1000 Sn 
Example 21 1 2 1 r
1000

R
Find two possible geometric sequences where the sum of the first two terms is 20 and the  2n  1  Use technology to produce a table:
sum of the first four terms is 1640, and write the general term of each sequence. 6
 2n  167.6 n 2n
S2  u1  u2  u1(1  r) 4
(1  r )
2
(1  r )(1  r )
2
Using the table, n = 8. 1 2
 S4 (1  r 4 )   (1  r 2 )
4
u (1  r )   (1  r)(1  r) (1  r 2 ) 2 4
S4  1  S2 (1  r)(1  r) When 8 or more terms are added, the sum
1 r  3 8

D
exceeds 1000.
S4 1640 4 16
  
1 r2 Find the ratio of S4 to S2.
S2 20 5 32
Simplify and solve for r.
 r2 81 6 64
 r 9 7 128
20 8 256
r 9  u1  2 Calculate u1 for each value of r.
10
or
20 5
r 9  u1  
8 2 Example 24 i

 x
The two possible geometric sequences are For what values of x does the series   1   , x ≠ −2 converge? Find the sum when x = −1.5.
i 1  2
given by the general terms:
2 3
 5  x  x  x
un 2  9 n 1
or un     9n 1 S  1    1  2   1  2   ... Write the first three terms of the series.
 2  2    
This is a geometric series where
 x
u1 r  1   Identify r.
 2

Continued on next page

24 25
1 FROM PATTERNS TO GENERALIZATIONS: SEQUENCES, SERIES AND PROOF 1.2
6 The first term of a geometric sequence is 9 12 The sum of n termsn of a certain series is

Number and algebra


For convergence: Use condition for convergence, ie −1 < r < 1. and the fifth term is 16. Show that there  1
given by Sn    2   1.
are two possible sequences and find their  
 x common seventh term.
−1 < 1 +  <1 a Find the first three terms of the series.
 2
7 The numbers 3a + 1,  a + 2, and a − 4 are b Show that the terms of the series are in
x three consecutive terms of a geometric geometric progression.
⇒ −2 < < 0

T
2 sequence. Find the two possible values of
⇒ −4 < x < 0, but x cannot be − 2 the common ratio.
13 The second term of a geometric series is 28
and the third term is 28(1 − a).
When x = −1.5, the series converges to
u
Use the formula S  1 . 8 The numbers a − 1,  a + 1,  and a − 2 are the Find the common ratio, given that the
1 r fourth, fifth and sixth terms, respectively,
0.5 1 series converges and the sum of the first
S   . of a geometric sequence. Find the common

F
1.5 3
three terms is 147.
ratio and the first term of the sequence.
14 A length of material measures 2 m. It is cut
9 Find the following series for the number of
into three lengths which are in geometric
terms stated:
1 1 progression. The longest piece is twice as
Example 25 a 3  1    ...
3 9
6 terms long as the shortest piece. Find the common

A
b 8 + 4 + 2 + 1 + … ratio of the sequence and the exact length
A geometric series converges to 8. The second term of the series is  5 . Find the common 10 terms
2 of the shortest piece.
ratio. c 0.1 + 0.03 + 0.009 + 0.0027 + …
15 terms 15 Write the first four terms of the series
i
u1 

 1   1 . Determine for what values
Use the formula S  . d 0.1 − 0.03 + 0.009 − 0.0027 + … x
u1

i
S  8  1 r
1 r  5 15 terms i 0  2 
  8(1  r)  r 

R
5 2 of x this series converges. Find the value of
u2  u1r    10 Calculate:
the series when x = −0.8.
2 6 n 1
a 7 3i
b  5  10i

16r  16r  5 
2
0 i 1 i 0

 (4r  1)(4r  5) 
0 Simplify and factorize.
11 Show that a geometric sequence with first
1
term 3 and seventh term has two
1 5 243
 r  or r 

D
4 4 possible sums to infinity and find them.

But, since the series converges, −1 < r < 1


1
Hence the answer is r   . TOK
4
Developing inquiry skills How do mathematicians
Go back to the original question about Koch’s snowflake and try to reconcile the fact that
Exercise 1C address the following, assuming that the length of each side of the some conclusions
original triangle is 81 cm: conflict with intuition?
• Calculate the perimeter of the snowflake at each iteration.
1 Write down the fifth term and the general 3 Determine the number of terms in each of • Calculate the area of the snowflake at each iteration.
term of each of the following sequences: the following sequences:
•  Tabulate the results and explain the number patterns that you observe.
a 1, 3, 9, … b 8, 4, 2, … a 0.02, 0.06, 0.18,  …,  3936.6 •  Create a model that helps you generalize the perimeter and area at any
x x3 x5 1 iteration.
c , , , ... d −3, 3, −3, 3, … b 64, 32,16, ...,
2 2 2 128
2 Determine the common ratio and write the 4 The fourth term of a geometric sequence is 6
terms indicated in each of the following and the seventh term is 48. Find the common Although it might not be obvious, you have actually been exposed
sequences: ratio and the first term of the sequence. to arithmetic and geometric sequences and series in previous
a 63, 21, 7, … 6th term 5 The third term of a geometric sequence mathematics classes. Usually this was in the form of solving word
81 27 is 6 and the fifth term is 54. Find the two problems. When we apply knowledge that was obtained by abstracting
b 243, , , ... 7th term generalizations of mathematical concepts to real-life situations we
2 4 possible values of the common ratio and the
a a a sixth term of each sequence.
c ,  , , ... 5th term
2 6 18

26 27
1 FROM PATTERNS TO GENERALIZATIONS: SEQUENCES, SERIES AND PROOF 1.2
are actually modelling the situation mathematically. The following
Students in group B decided that the data could be modelled using a

Number and algebra


investigations illustrate how arithmetic sequences can be hidden in
everyday practices. geometric sequence by taking an average common ratio.
1 Create each growth model and determine the number of smartphones
shipped in the years 2014 to 2016 as predicted by each model.
Investigation 8 2 The actual shipments for the years 2014 to 2016 are given below. Which

T
Before the start of the school year, a stationer needs to stock up with seems to be the better model for growth?
notebooks. He has been given the following offers:
Year Shipments worldwide in billions
Provider A: Notebooks in packs of 20, at an offer of 6 for the price of 4, where
each packet of 20 notebooks costs €10. 2014 1.32
Provider B: Notebooks in packs of 100, at an offer of 3 for the price of 2, where 2015 1.46

F
each pack of 100 costs €48. 2016 1.51
The stationer is considering stocking between 500 and 3000 notebooks, in 3 U
 se your models to predict shipment numbers up to and including the
multiples of 100. year 2025.
1  Factual The stationer first looks at the offer made by Provider A and
realizes that he would get the cheaper rate when the number of notebooks 4  Conceptual How can you determine whether an arithmetic series grows

A
ordered are in a particular arithmetic sequence. Show that the stationer is faster than a geometric series? Which model looks more realistic and
correct. why?

2  Factual The stationer’s wife tells him that the argument is true for the
offer from Provider B. Is the wife also correct? The following examples illustrate how arithmetic and geometric
sequences and series can be used to solve problems.
3  Conceptual They then compare costs incurred when buying notebooks

R
from Provider A and from Provider B. They notice that for certain numbers
of notebooks ordered it would be cheaper or the same rate if they were to
order from Provider A. Determine which numbers they are referring to. How Example 26
is this list of numbers different from the previous two answers? The number of Facebook users at the end of 2008 was 145 million and growing at a rate of
4  Conceptual The stationer would like to divide his order of 1500 3% per week. At the end of 2010 the number of Facebook users was 608 million.

D
notebooks between the two providers. How can he divide up his order to a If the rate of growth had remained constant at 3% per week, determine the number of
minimize his cost price? If he sells the notebooks at 45 cents each, what users at the end of 2010.
would be his percentage profit? How does this compare to his percentage
b 
The growth rate of 3% per week remained steady for 6 months and then dropped to 1.1%
profit had he ordered all the notebooks from either of the providers?
per week. This growth rate was maintained for another six months but then it dropped to
5 How would this work help the stationer when making the order? 0.75% per week. Assuming that this rate was sustained for a whole year, show that this
model better describes the recorded numbers.
6 How realistic is the selling price fixed by the stationer?
c If the rate of growth dropped to 0.6% at the beginning of 2011 and remained steady,
determine how long it would take for the number of users to reach 1 billion.

Investigation 9 a The number of users after 2 years is A growth of 3% weekly can be modelled
145 × (1.03)104 ≈ 3136 million. using a geometric sequence where the
Students were given the following data and asked to work in groups to create value after 2 years (104 weeks) is given by
a growth model for the shipment of smartphones worldwide from 2011 to b The number of users after 6 months is
145 × (1.03)104.
2016. 145 × (1.03)26 ≈ 313 million.

The number of users at the end of 2009 All answers are given to 3 significant
Year Shipments worldwide in billions
is 312.706 × (1.011)26 ≈ 416 million. figures.
2011 0.52

The number of users at the end of 2010
2012 0.74
is 415.59 × (1.0075)52 ≈ 613 million.
2013 1.05
Students in group A decided that the data could be modelled using an Continued on next page
arithmetic sequence by taking the average difference.

28 29
1 FROM PATTERNS TO GENERALIZATIONS: SEQUENCES, SERIES AND PROOF 1.2

Number and algebra


c 
The number of users at the end of 2010  1 − (1.03)7 
Number of Number of users = 12 298.73 + 200 
is 613 million and the growth model is  = 12 298.73 + 200(7.662..)
Year years 613 × 1.006n  1 − 1.03 
given by 613 × (1.006)n.
2011 1 616.678 = $13 831 (to the nearest dollar)
We want 613 × (1.006)n = 1000.
2012 2 620.378068

Using technology and table to solve for n Money invested without interest = 10 000 + 200 × 7 = 1140

T
gives: 2013 3 624.1003364
Total interest = 13 831 − 1140 = $2431

The number of users will reach 1 billion 2014 4 627.8449384
during the year 2092, which is 82 years 2015 5 631.6120081
after the end of 2010. 2016 6 635.4016801
Or use GDC to solve the equation.
Exercise 1D

F
. . .
. . . 1 In 2010, a shop sold 220 televisions. Every six She has 1.5 kg of sugar available and
. . . months the shop sold five more televi-sions, knows that the amount of sugar needed is
so that it sold 230 televisions in 2011, 240 in usually half the amount of flour by weight.
2090 80 989.2337664 2012, etc. After the first two trials she decides to
2091 81 995.169169 change the amount of sugar in each

A
a Evaluate how many televisions the shop
2092 82 1001.140184 sold in 2017. trial according to a geometric sequence.
Evaluate the number of trials this would
b Calculate the total number of televisions
allow her to carry out. Explain how
sold between 2010 and 2017 inclusive.
reliable her model is for using a geometric
c The selling price of a television was €600 growth model for sugar content.
in 2010 but the selling price fell by €20
Example 27

R
each year. In a particular year the number 5 The first diagram shows a sequence of
When Jacob turned 18 he had access to the money his grandparents had invested in a of televisions sold by the shop was half squares.
savings account. He decided to reinvest $10 000 at a compound interest rate of 3% each of the selling price of each television. Starting with the largest square, the
year. He decided to add $200 to this investment on his next birthday and each following Determine in which year this occurred. midpoints are joined to form the second
birthday until he turned 25. Evaluate how much money was in his account just after his square of the sequence. This process can be
2 Jane started working in 2000. On each
25th birthday. Evaluate the total interest gained over this time. continued infinitely (in theory, but not in

D
successive year she received a salary increase
equivalent to 1.5% of her previous salary. In practice!)
On his 18th birthday Jacob had $10 000. Amount in bank + interest for
2011 her salary was €49,650. Determine her
one year.
Just before his 19th birthday he had 10 000(1.03) starting salary to the nearest €.
Value after 1 year + $200.
On his 19th birthday the amount was 10 000(1.03) + 200. 3 Carla traced her family tree back four gener­
ations. Carla’s parents are the first generation
Just before his 20th birthday: (10 000(1.03) + 200) ×
back and her first set of ancestors. Carla’s four
1.03
grandparents are the second generation back
Just after his 20th birthday: and her second set of ancestors.
(10 000(1.03) + 200) × 1.03 + 200 a How many ancestors are in Carla’s
family tree?
= 10 000(1.03)2 + 200(1.03)1 + 200
b Determine how many generations back
After 7 years, just after his 25th birthday the amount Using pattern from years 1 and 2. she would have to trace to find more
would be: than 1 million ancestors.
10 000(1.03)7  200(1.03)6  200(1.03)5  ...  200 4 Prisana is testing a recipe for a cake and
tries it out several times, adjusting the
 10 000(1.03)7  200((1.03)6
  (1.03)5
 ...  (1.03) 1)
 Geometric series with u1 = 1,  
amount of flour and sugar used each time.
Geometric series r = 1.03 and n = 7.
In the first recipe she uses 200 g of flour,
and she decides to increase the weight of
flour by 20 g in each trial. She has time to
try out the recipe just 10 times. How many
kg of flour would she need?

30 31
1 FROM PATTERNS TO GENERALIZATIONS: SEQUENCES, SERIES AND PROOF 1.3
a If the sides of the largest square have a each row is 15 cm high and 95 cm deep. c Evaluate how much more money he a Construct a model to represent the

Number and algebra


length of 2 units, calculate the side lengths Assuming that the front row is 3 m away would have made if he had divided up amount of drug in the bloodstream at
of the second, third and fourth squares. from the screen and the ceiling at the back the $5000 equally between the savings the end of day 1, ie immediately after
A spiral is formed by joining segments row is 2.4 m high, determine: account and bonds at the very start? the fourth administration.
shown as red lines in the second diagram. (Give all your answers to the nearest b The company advises that the drug should
b the horizontal distance from the screen
dollar.) not be administered for more than 10 days.
b Use your answers in part a to find the to the top row

T
length of the spiral shown. 10 A pharmaceutical company has developed Evaluate the maximum amount that should
c the maximum height of the ceiling.
a drug that fights a bacterial infection. be administered to ensure that the amount
c Explain what happens to the length of the 8 Ayla, Brynna and Cindy each receive of drug in the bloodstream does not exceed
spiral if we continue the process infinitely. The drug is to be administered four times
€200 from their parents with the TOK per day, every six hours. It was found the safety level?
A different spiral is formed by shading the condition that they promise to invest that six hours after administering, 37.5% c The drug starts being effective when the
triangles as shown in the third diagram. Do all societies

F
it for at least 10 years. Ayla invests of the original amount was still in the amount of drug in the bloodstream is
d Find the total area of the shaded her money in an account with Rapid view investment bloodstream. The maximum safe level of 7 mg ml-1. Determine how many times
Bank that offers 5% simple interest and interest in the
triangles. the drug in the bloodstream is 8 mg ml-1. the drug must be administered for this
annually. Brynna says it is better same way?
e Determine the total area of the spiral level to be reached.
to invest in an account that offers What is your
formed if the process of forming squares compound interest because it grows stance?

A
and shading triangles is continued quicker, so she uses an account with
infinitely.
6 Gyuhun takes out a loan of $1500 to
Quick Bank that offers 3.5% interest
compounded annually. Cindy is not sure Developing inquiry skills
furnish his new student apartment. which offer is best so she invests €100 with You should now go back to the start of the chapter and answer the
The terms of the loan are that Gyuhun Rapid Bank and €100 with Quick Bank. questions in the scenario that involves taking a loan for buying a new car.
will pay equal monthly instalments. Answer the following questions giving your

R
Interest is calculated monthly and is answers to the nearest €.
a Evaluate how much each investment is
charged at 12% p.a. The loan is to be
repaid in two years. worth after 10 years. Developing your toolkit
a Calculate the amount that Gyuhun b How much is each investment worth Now do the Modelling and investigation activity on page XX.
has to pay each month if the first after 25 years?

D
repayment is made one month after the c Determine after how many years the
money is borrowed and after interest is
calculated.
three investments yield approximately
the same amount.
1.3 Proof
b Evaluate how much, to the nearest 9 At the start of 2010 Karim had $5000 to
dollar, Gyuhun actually has to pay for invest. He decided to invest part of the
Investigation 10a
furnishing the apartment? money in a savings account that offered Each of the three diagrams below, not drawn to scale, consists of a square ABCD of different sizes. Line PR
1.5% simple interest per year. He added is perpendicular to AB and DC and line SQ is perpendicular to AD and BC. Copy and complete the table on the
HINT $1000 to this amount and fixed it for 10 next page.
Per annum (p.a.) is a term often years in bonds that offered 2.5% compound A 4 P 3 B A 8 P 3 B A 7 P 5 B
used in financial contexts and interest per year. The rest of the money he
means “for each year”. invested in shares.
a After one year the money invested in
7 An architect is designing a cinema that shares made a loss of 1%. Given that
should hold 570 seats. The front row of
the total amount of money invested
the cinema can hold 30 seats and each 4 8 7
increased by $75, determine how much
consecutive row is to hold six seats more
money Karim invested in each. T
than the previous row. T T S Q
b At the end of the first year, Karim S Q S Q
a Evaluate how many rows the cinema decided to sell the shares at their current 3 3 5
can hold. value and reinvest the money in the
For optimal viewing, the floor of the cinema savings account. Evaluate the total value D R C   D R C   D R C
needs to be stepped so that the step between of his investment at the end of 2020.
Continued on next page

32 33
1 FROM PATTERNS TO GENERALIZATIONS: SEQUENCES, SERIES AND PROOF 1.3
3  Factual What do you call each of these relationships? Why?

Number and algebra


Area ABCD Area APTS Area BPQT Area STRD Area TQCR A a P b B

(3 + 4)² = 49 4² 3×4 4 Determine the value of x in each case.


x
5 Now rewrite the relationship for the diagram on the right. Q
6 Factual What do you call this relationship? Why? x

T
1 D
 escribe the relationship between the areas of A a P b B
square ABCD, rectangles PBCT and TRDS, and square x
TQCR. S
x
2 Rewrite the relationship above replacing words with
numbers.

F
a D R C
3 Now rewrite the relationship for the diagram on the
right.
4  Factual What do you call this relationship? Why? S
T
Q What is proof? TOK
b In Investigation 10a, you should have noticed that by making the ques­

A
What is the role of
tion visual you were able to prove the identity (a + b)² ≡ a² + 2ab + b².
the mathematical
By looking at a square which is divided into two smaller squares and
D R C community in
two rectangles and comparing areas you came to a valid conclusion, and
determining the validity
by then representing numbers with variables you could show that this
of a mathematical
identity is valid for all values of a  and b. We can say that you have proved
proof?
Investigation 10b the statement (a + b)² = a² + 2ab + b² is true for all a, b ∈ + because the

R
perpendiculars PR and SQ could be placed anywhere along the sides of
Each of the three diagrams below, not drawn to scale, consists of two squares ABCD and PQRS. square ABCD, which could also be as large or as small as you wanted.
Use this information to copy and complete the table for each diagram.
Similarly, in Investigation 10b, you were able to validate the statement
A 3 P 4 B A 5 P 12 B A7P 24 B that in any right-angled triangle the lengths of the sides obey the
5
7 relationship a² + b² = c², where c is the length of the hypotenuse of the
3 x x Q

D
x Q triangle.

x Q
4 12
x x
A proof in mathematics often consists of a logical set of steps that validates
24
the truth of a general statement beyond any doubt.
x x
S 4 12 24
x There are many ways of presenting a proof and we will be looking at
x S
3
x S x some of these ways in this section.
5 7
D 4 R 3 C   D 12 R 5 C   D 24 R7C Types of proof
Area ABCD Area PQRS Area ∆PBQ Area PQRS + 4 × area ∆PBQ
Investigation 11
(3 + 4)² = 49 x² 6
Copy and complete the table below and then suggest a conjecture.

x² 1
1+3
1 Describe the relationship between the areas of the squares and the triangles.
1+3+5
2 Rewrite the relationship above replacing words with numbers and making the area of 1+3+5+7
PQRS the subject.
Continued on next page

34 35
1 FROM PATTERNS TO GENERALIZATIONS: SEQUENCES, SERIES AND PROOF 1.3
Example 28

Number and algebra


1+3+5+7+9
1 + 3 + 5 + 7 + 9 + 11 Show that 1 + 3 + 5 + 7 + … + (2n − 1) = n².


S 1  3 5  ...  (2n  3)  (2n  1) Write out the sum in reverse
S  (2n  1)  (2n  3)  (2n  5)  ...  3  1 order.

T
2S 2n  2n  2n  ...  2n  2n
 Add the two sums and simplify.
n times

Now let’s look at the same sequence visually.  2S 2n²


Below you will find two visuals that represent a 4 × 4 square differently. In the first one, if you add the S n²
squares shaded white and green alternately you will obtain the sequence 1 + 3 + 5 + 7 = 16.

F
A direct proof is a way of showing the truth of a given statement by constru­
cting a series of reasoned connected established facts. In a direct proof the

A
following steps are used:
• Identify the given statement.
• Use axioms, theorems, etc, to make deductions that prove the conclusion
of your statement to be true.

R
The proof given in Example 28 consists of a set of reasoned steps
that leads to the required result. Note that we did not just quote
the sum of an arithmetic sequence with first term 1 and common
difference 2, although by the definition of direct proof this would
The second visual is made up of the sequences 1 + 2 + 3 + 4, and the sequence 1 + 2 + 3, and when placed have been valid.
next to each other in the orientation shown they form a 4 × 4 square giving a sum of 16.

D
4 3 2 1
Example 29
Show that:
4 3 2 a The sum of an odd and even positive integer is always odd.
1
b The sum of two even numbers is always even.
4 3 c The sum of two odd numbers is always even.
2 2
a 
Let m  and n  be an odd and an even An odd positive integer can be written as
4
positive integer respectively. 2k − 1  where k ∈ +.
3 3 3
⇒ m = 2p − 1  and n = 2s where p, s ∈ + An even positive integer can be written as
⇒ m + n = 2p − 1 + 2s = 2(p + s) − 1 2k  where k ∈ +.
Combining the two visuals we can say that:
This is an odd number since p + s ∈ +.
4²  1  3  5  7
b 
Let m  and n  be two even positive
 (1  2  3  4)  (1  2  3) integers.
4 3 ⇒ m = 2p  and n = 2s where p, s ∈ +
 (1  4)  (1  3)
2 2
⇒ m + n = 2(p + s)
 10  6
This is an even number since p + s ∈ +.
Seeing the question from this perspective should make your conjecture more valid. However, this alone is
not a proof, because you cannot really generate a general square. A more rigorous proof of the conjecture is Continued on next page
required and this is found in Example 28.

36 37
1 FROM PATTERNS TO GENERALIZATIONS: SEQUENCES, SERIES AND PROOF 1.3
Example 32

Number and algebra


c 
Let m  and n  be two odd positive integers.
⇒ m = 2p − 1  and n = 2s − 1  where  Prove that if the sum of the digits of a four-digit number is divisible by 3, then the four-digit
p, s ∈ + number is also divisible by 3.
⇒ m + n = 2p + 2s − 2
Let n be a four-digit number such that
⇒ m + n = 2(p + s − 1)

T
n = a3a2a1a0.

This is an even number since
p + s − 1 ∈ +. We know that:
a3 p  10³  Since we are given that n is a four-
 digit number, p ≠ 0.
a2 q  10² 

F
 0  p, q, r , s  9and p  0
a1  r  101 
Example 30
2 2 a0  s  100 
  a   a 
Show that  x         x ²  ax .
  2   2  and p + q + r + s = 3k, k ∈ . We are given that the sum of the
digits is a multiple of 3.
⇒ n = p × 10³ + q × 10² + r × 101 + s × 100

A
 a a ²  a²
LHS   x ²  2   x    Expand and simplify.
  
2 4 4 and s = 3k − p − q − r
 x ²  ax Therefore
n = p × 10³ + q × 10² + r × 101 + 3k − p − q − r
= (p × 10³ − p) + (q × 10² − q) + (r × 101 − r) + 3k

R
In Example 30 we started from the left hand side and showed that this
is equivalent to the right hand side. When writing down a proof it = p(10³ − 1) + q(10² − 1) + r(101 − 1) + 3k
is very important to work on one side of the statement only. A
proof is also valid if you work on each side consecutively to obtain the = 999p + 99q + 9r + 3k
same result. This method is shown in the example below. = 3(333p + 33q + 3r + k)  
Since (333p + 33q + 3r + k) ∈  it follows that n is

D
Example 31 divisible by 3.

Prove that (n + 4)2 − 3n − 4 = (n + 1)(n + 4) + 8.

LHS Note that you work on each side separately


and not in the same line.
Example 33
= n2 + 8n + 16 − 3n − 4 1 1 1 1 1 1 1
Show that       ... .
= n2 + 5n + 12 2 4 8 16 32 64 3

RHS LHS Separate the given series into one


1 1 1  1 1 1  series with positive terms and one
= n2 + n + 4n + 4 + 8      ...       ...  with negative terms.
 2 8 32   4 16 64 
= n2 + 5n + 12  1 1  1  1  1 2   1 1  1  1  1 2  Note that this is the difference of two
         ...           ...  converging geometric series.
LHS = RHS 2 24 24  4 44 44 
     
Therefore Converging geometric series Converging geometric series Use the formula for sum of converging
1 1 geometric series and simplify.
(n + 4)2 − 3n − 4 = (n + 1)(n + 4) + 8
2 4 2 1 1
    
1 1 3 3 3
1 1
4 4

38 39
1 FROM PATTERNS TO GENERALIZATIONS: SEQUENCES, SERIES AND PROOF 1.3
Exercise 1E Investigation 13

Number and algebra


1 Prove that (a + b)2 + (a − b)2 = 2(a2 + b2). 8 The diagram here shows a trapezium ABCD In geometry, the triangle inequality states that in any triangle ABC, the B

that has been divided into three triangles. sum of the lengths of any two sides must be greater than or equal to the
2 Show that the product of two odd numbers length of the remaining side.
Use your knowledge of areas and the c
is always an odd number.
diagram to show that a2 + b2 = c2. Applying this to the triangle on the right we get the following:

T
a
3 Prove that a four-digit number is divisible A a B a + b ≥ c
by 9 if the sum of its digits is divisible by 9. a + c ≥ b
A

Hence identify which of the numbers 3978, b + c ≥ a


5453, 7898, 9864, 5670 are divisible by 9 b b

without carrying out any division. c


1 Explain the situation when:

F
a+b=c C
4 Show that E
a+c=b
(a2 + b2)(c2 + d2) = (ad + bc)2 + (bd − ac)2. A
b+c=a 3
1 2 1 2 1 2 1
5 Prove that       ... .
3 9 27 81 243 729 8 2 Look at triangle ADC in the quadrilateral on the right and write the 2 B
6 Prove that the difference between the triangle inequality in terms of AD and DC.

A
a c
squares of two consecutive numbers is
3 Now apply the triangle inequality to triangle ABC in terms of AC D
always an odd number.
and AB.
7 Show that 1 1 1 n2  1 . Hence
   2 4 Look at the inequalities obtained in questions 2 and 3 and
(n  1) n (n  1) n(n  1)
1 1 1
comment on your result.
determine the value of   . D b C 10
5 6 7 5 Now draw diagonal BD and repeat steps 3 and 4 and 5 for

R
4
triangles ABD and DBC.
6 What happens if you change the order of the sides?
Proof by contradiction 7 Factual What do you conclude from this investigation?
8  Conceptual How else could you have come to the same
Investigation 12

D
conclusion?
C
You will now look at the statement below and answer the questions:
If n∈ and 5n + 2 is even, then n is even.
1 Write a direct proof of this statement. TOK
When setting out a proof by contradiction you follow the following steps:
2 Now assume that n is an odd number and rewrite the expression 5n + 2 to
• Identify what is being implied by the statement. What do mathematicians
reflect this.
• Assume that the implication is false. mean by mathematical
3 Simplify your expression and explain why this can never be an even HINT proof, and how does it
number. • Use axioms, theorems, etc … to arrive at a contradiction.
A proof by contradiction differ from good reasons
4 Factual How is the second method different to a direct proof? is built on the logical • This proves that the original statement must be true. in other areas of
reasoning that If a knowledge?
proposition is true then In other words, the assumption contradicts either a given statement or
its contrapositive is something we already know to be true, or in some cases both.
In Investigation 12 you managed to find a different argument to prove also true. That is, if in
the statement by using the contrapositive. You started by assuming a statement a implies
that the second part of the statement is false and showed that this b then it is also true International-mindedness
led to a contradiction, ie the first part of the statement was also false. that if b is not true, this Reductio ad absurdum (reduce to absurdity) is a term used to describe
In this case, the statement could easily be proved directly but this is implies that a is not logical reasoning that attempts to disprove a statement by showing that
not always the case, and sometimes you will need to use the second true.
method to prove a statement correct. it leads to an absurd result. In fact, this method can be traced back to
the Greek philosopher Aristotle where he talks about “reduction to the
impossible” in his book Prior Analytics.

40 41
1 FROM PATTERNS TO GENERALIZATIONS: SEQUENCES, SERIES AND PROOF 1.3
The following examples will help you understand this method of proof

Number and algebra


so that you can then apply it in set tasks. But it was assumed that m, n
have no common factors and now
we have found that they have a
Example 34 common factor of 2.
Prove by contradiction: Hence, we cannot find m, n ∈  that

T
have no common factors to make
a if the integer n is odd then n2 is also odd m
2  a rational number.
b if n2 is even then n is also even n
a Assume that n2 is even. Start by making an assumption that the
⇒ n2 = 2k, k ∈  resulting statement is false.

F
Example 36
⇒ n × n = 2k 1
Prove that there is no x ∈  such that  1  x.
But this cannot be true if n is odd as x 2
we know that the product of two odd Assume there exists a real number a such Assume that a real solution x = a exists.
numbers is also odd. 1
 1  a.

A
that Solve for a.
Hence given that n is odd n2 is also odd. a2
⇒ 1 = (a − 2)(1 − a) Apply the quadratic formula and conclude
b Assume that n is an odd integer. We want to prove that n2 even ⇒ n even. that a is not a real number since the
⇒ 1 = −a2 + 3a − 2
⇒ n = 2k ± 1, k ∈  To use the contrapositive we start with n number under the square root is negative.
and deduce the contradiction ie that n2 is ⇒ a2 − 3a + 3 = 0
⇒ n2 = (2k ± 1)2 = 4k2 ± 4k + 1 Show that a cannot be a real number.
not even. 3  9  12

R
⇒ n2 = 2(2k2 ± 2k) + 1 = 2p + 1 a ∉ 
2
Which is an odd number.
Hence given that n2 is an even integer n
is also an even integer. Example 37

D
Prove that if m, n ∈ , then m2 − 4n − 7 ≠ 0.
  

International- Assume that m2 − 4n − 7 = 0. Assume that the statement is false.


Example 35
mindedness ⇒ m2 = 4n + 7
Show that 2 is irrational.
How did the ⇒ m2 = 4n + 6 + 1 Rearrange the equation.
m
Assume that 2 where m, n ∈  Make the assumption Pythagoreans find out ⇒ m2 = 2(2n + 3) + 1 Since n ∈  ⇒ 2n + 3 ∈ .
n that the statement is
and m, n have no common factors. that 2 is irrational? This means that m2 is an odd integer.
2
false, ie 2 is rational.
m But this means that m is an odd integer since an
 2 2
n even integer squared is even.
m  2
2n2 Let m = 2p + 1 where p ∈ .
This means that m is an even integer. Here we are using the Then
fundamental theorem of (2p + 1)2 − 4n − 7 = 0
⇒ m = 2k  and m2 = 4k2
arithmetic which states
This leads to: that every integer bigger ⇒ 4p2 + 4p + 1 − 4n − 7 = 0
than 1 is either prime ⇒ 2p2 + 2p − 2n − 3 = 0
m2 = 4k2 = 2n2
or a multi­ple of primes
⇒ 2(p2 + p − n) = 3
⇒ n2 = 2k2 and since m is an even
integer it must contain a Since p, n ∈  this cannot be true as 3 is not even.
This means that n is an even integer.
prime factor of 2.
Hence m2 − 4n − 7 ≠ 0 given that m, n ∈ .

42 43
1 FROM PATTERNS TO GENERALIZATIONS: SEQUENCES, SERIES AND PROOF 1.3
Sometimes we encounter statements that seem to be true, and Proof by induction

Number and algebra


for every example that we consider the statement seems to
hold. Although such examples are good to verify a statement or
conjecture, they are not sufficient to prove the statement. It takes Investigation 14
only one example that contradicts the statement to justify that the
Look at the diagrams and answer the questions.
statement is wrong.

T
A counterexample, or counterclaim, is an acceptable “proof” of the fact that a
given statement is false.

Some examples follow to demonstrate how counterexamples can be used.

F

Example 38 1 Each of the three diagrams represents a series. Write them down.
Show by a counterexample that the following statements are not true.
2 If the diagrams were to continue, what would the next three diagrams be?
a If n ∈  and n2 is divisible by 4, then n is divisible by 4.

A
3 Write a conjecture based on your findings.
b If n ∈  then n2 + 1 is a prime number.
c If an integer is a multiple of 10 and 12 then it is a multiple of 120. 4 Prove your conjecture using a direct proof.

a 
When n = 6, n2 = 36 which is divisible by Again, the statement is true for many integers
4, but 6 is not divisible by 4. but we only need one counterexample. In Investigation 14, you used a visual representation of a series to make a
b 
n = 3 ⇒ n2 + 1 = 10 which is not a prime

R
conjecture about a special series and then prove it. In this case, you were
number. able to prove the conjecture directly, but there are times when
c 
60 is a multiple of both 10 and 12 but it such a direct proof is not possible. Sometimes we need to
is not a multiple of 120. revert to a different proof which is called proof by induction.
To illustrate the principle of proof by induction, imagine two
dominoes placed standing at a distance less than half their

D
Exercise 1F length, as illustrated in the diagram on the right. If the first
domino is knocked over it will fall and cause the second
In questions 1 to 9, prove the statements by 9 Given that m and n are positive integers, it domino to fall with it. This is the starting point of
contradiction. follows that m2 − n2 ≠ 1. the process and is called the basic step.

10 Show by a counterexample that the Now assume that the domino in the kth place falls if the
1 For all n ∈ , if n2 is odd then n is also odd. domino before it (in the (k - 1)th position) falls. This
following statements are not true:
2 3 is irrational. assumption is the second step in the process.
a (m + n)2 ≠ m2 + n2
5
3 2 is irrational. b If a positive integer is divisible by a If we were to add another domino at the end of the k dominoes,
prime number then the number is not this last domino will also fall. This analogy represents the final
4 For all p, q ∈ , p2 − 8q − 11 ≠ 0. step of the process which is called the inductive step.
prime.
5 For all a, b ∈ , 12a − 6b ≠ 0. We can then finalize our argument by stating that at the start
c 2n − 1 is a prime number for all n ∈ .
6 
If a, b, c ∈ , where c is an odd number and d 2n − 1 is a prime number for all n ∈ +. of the process it was shown that the first domino caused the
a2 + b2 = c2, then either a or b is an even second domino to fall. We can now use the second and third step that a
e The sum of three consecutive positive third domino placed behind the first step will also fall, and again using
number.
integers is always divisible by 4. the two steps, a fourth domino will also topple over. We can continue
7 If n, k ∈ , then n2 + 2 ≠ 4k. f 
The sum of four consecutive positive repeating this process as many times as we want. In other words, we
8 
If r  is an irrational number and q is a integers is always divisible by 4. have shown that we can have as many dominoes as we like and they
rational number then p + q is also irrational. will all fall if the first domino knocks over the second domino.
We apply the dominoes analogy to mathematics to prove the statement
in Investigation 14. It should be noted that the visual could have
started from a previous step, ie with just one green circle.

44 45
1 FROM PATTERNS TO GENERALIZATIONS: SEQUENCES, SERIES AND PROOF 1.3
In Example 39, we are going to use this as the basic step, so that we We then move to the inductive step 10k+1 = 10 × 10k and using the
assumption would give 10k+1 = 10 × 10k = 10 × 7a = 7(10a). Since a

Number and algebra


have the proof for all positive integer values of n.
is a positive integer, 10a is a positive integer also, so 10n is a multiple
of 7 for all positive integers. Of course, we know that this is not
Example 39 true because 10 = 2 × 5 ⇒ 10n = 2n × 5n and since 2 and 5 are prime
numbers 7 will never divide 10n exactly.
Prove by mathematical induction that 1 + 2 + 3 + … + (n − 1) + n + (n − 1) + … + 3 + 2 + 1 = n2.

T
Incorrect use of the inductive step
P(n): 1 + 2 + 3 + … + (n − 1) + n + (n − 1) + We start by making a statement P(n) which Proof by induction is also not valid if the assumption is not used in the
… + 3 + 2 + 1 = n2, n ∈ + we need to prove true for certain values of n inductive step, as shown below to prove that 11n − 6 is a multiple of 5.
(usually positive integers).
P(n): 11n − 6 = 5a, where  n, a ∈ +.
When n = 1

F
This is the basic step.
Basic step:
LHS =1
When n = 1,   LHS = 11 − 6 = 5.
RHS =1
Therefore, the statement is true for n = 1.
Since LHS = RHS ⇒ P(1) is true This is where we make the assumption.

A
Note the wording just before we substitute Assumption:
Assume that P(n) is true for some value of
for n in the statement. Assume that the statement is true for some k ∈ +, k ≥ 1.
k,  k ≥ 1, k ∈ + ie
1 + 2 + 3 + … + (k − 1) + k + (k − 1) + … + 3 ie 11k − 6 = 5b, where  n, b ∈ +.
+ 2 + 1 = k2
Inductive step:
When n = k + 1 LHS In the inductive step we must use the When n = k + 1:

R
assumption to show that the statement is
= 1 + 2 + 3 + … + (k − 1) + k + (k + 1) + k + LHS
also true for k + 1.
(k − 1) + … + 3 + 2 + 1
= 11k+1 − 6
= 1 + 2 + 3 + … + (k − 1) + k + (k − 1) + … +
= 11 × 11k − 6
3 + 2 + 1 + (k + 1) + k
= (5 + 6) × 11k − 6 Write 11 as 5 + 6
= k2 + (k + 1) + k

D
=5× 11k +6× 11k − 6 Distribute 11k
= k2 + 2k + 1 HINT
= 5 × 11k + 6(11k − 1) 6 is a common factor
= (k + 1)2 In the chapter review at the end of this
= 5 × 11k + 6(11 − 1)(11k−1 + 11k−2 + 11k−3 + … + k + 1) chapter you will be guided to prove that
Since P(1) was shown to be true and it This final statement completes the proof
was also shown that if the statement is and should always be included. = 5 × 11k + 60(11k−1 + 11k−2 + 11k−3 + … + k + 1) an − bn = (a − b)(an−1 + an−2b +
true for some n = k, k ∈ +, it is also true an−3b2 + … + abn−2 + bn−1)
= 5(11k + 12(11k−1 + 11k−2 + 11k−3 + … + k + 1))
for n = k + 1, it follows by the principle of
mathematical induction that the statement = 5m
is true for all positive integers. This is the result required and the mathematics above is correct.
However, the assumption was not used to obtain the result. Hence
the proof by mathematical induction is incorrect. The correct
solution is shown in Example 40.
Why is the basic step important?
The principle of mathematical induction is very rigorous, provided that
all the steps are used. If the basic step is left out, we can end up with Example 40
erroneous results. Suppose you were asked whether 10n is a multiple Use mathematical induction to prove that 11n − 6 is a multiple of 5.
of 7 and you try using mathematical induction without the basic step.
Here is what you would obtain: P(n): 11n − 6 = 5a, where  n, a ∈ + Write the statement that you want to prove.
Assume 10k = 7a for some n, a ∈ +. When n = 1 Basic step, show statement is true for n = 1.

Continued on next page

46 47
1 FROM PATTERNS TO GENERALIZATIONS: SEQUENCES, SERIES AND PROOF 1.3

Number and algebra


LHS = 11 − 6 = 5 Sk 
1 Sk  uk 1 Use assumption to obtain result
Therefore, the statement is true for n = 1. k k 1
  2u1  (k  1)d   u1  kd 
Sk 1  2u1  ((k  1)  1)d .
2 2
Assume that the statement is true for some Make assumption and then make 11k the k
k ∈ +, k ≥ 1. subject of the formula.  ku1  (k  1)d  u1  kd Use algebra to simplify.
2

T
ie 11k − 6 = 5b, where  n, b ∈ +  k 1 
 u1(k  1)  kd   1
 2 
⇒ 11k = 5b + 6
 k 1
When n = k + 1: Inductive step.  u1(k  1)  kd  
 2 
LHS  k  1

F
  (2u1  kd)
 = 11k+1 − 6 Let n = k + 1.  2 
Rewrite the LHS in terms of 11k.  k  1
11  11k  6   (2u1  ((k  1)  1)d )
 2 
 11(5b  6)  6 Use the assumption.
Since P(1) was shown to be true and it Write the final statement.
 55b  66  6 Simplify to obtain the required result, was also shown that if the statement is

A
 55b  60 in this case write it as a product with a true for some n = k, k ∈ +, it is also true
factor of 5. for n = k + 1, it follows by the principle
 5(11b  12)
of mathematical induction that the
Since P(1) was shown to be true and it was also Write final statement.
statement is true for all positive integers.
shown that if the statement is true for some n
n = k, k ∈ +, it is also true for n = k + 1, it follows u1(1  r )
b P(n): Sn 
1 r

R
by the principle of mathematical induction that
the statement is true for all positive integers. LHS: When n = 1,  S1 = u1 Basic step.
u (1  r)

RHS: When n 1,
1
u1
1 r
LHS = RHS
Example 41 Therefore P(1) is true. Assumption.

D
Prove the following statements using mathematical induction.
Assume that for some k ∈ + Substitute k in the statement.
a 
The sum of the first n terms of an arithmetic sequence with first term u1 and common u1(1  r )
k
n Sk 
difference d is given by 
Sn  2u1  (n  1)d . 1 r
2
b The sum of the first n terms of a geometric sequence with first term u1 and common ratio When n = k + 1 Inductive step.
u1(1  r )
n
Sk  Sk  uk 1 Use assumption to obtain result.
r is given by Sn  . 1

1 r u1(1  r )
k
  u1r k
n 1 r k 1
a P(n): 
Sn  2u1  (n  1)d  k
u1(1  r )  u1r (1  r)
k Sk 1 
u1(1  r )
.
2 1 r

LHS: When n = 1,  S1 = u1 Basic step. 1 r Use algebra to simplify.
k k k 1
1 u1(1  r  r  r )

RHS: When n 1, (2u1  (1 
1)d) u1 
2 1 r
LHS = RHS u1(1  r
k 1
)
Therefore P(1) is true. Assumption. 
1 r
k
Assume that 
Sk  2u1  (k  1)d  Substitute k in the statement. Since P(1) was shown to be true and it Write the final statement.
2 was also shown that if the statement is
for some k ∈ +. true for some n = k, k ∈ +, it is also true
When n = k + 1 Inductive step. for n = k + 1, it follows by the principle
of mathematical induction that the
statement is true for all positive integers.

48 49
1 FROM PATTERNS TO GENERALIZATIONS: SEQUENCES, SERIES AND PROOF 1.4
n
3 Given the statements below, decide
Example 42 c  2 i
2n  1

Number and algebra


i 0
how best you can prove or disprove the
Use mathematical induction to prove that 32n + 7 is divisible by 8 for all n ∈ . statement. Some statements can be proved
d 9n − 1 is divisible by 8 for all n ∈ . in more than one way; you should attempt
P(n): 32n + 7 = 8A n2(n  1)2 both ways and then decide which is the
Note that since the statement holds for n ∈  e 13  23  33  ...  n3 
4 more elegant proof, and why.
When n = 0, we have to start with 0.

T
f n − n is divisible by 3 for all n ∈ .
3 a Prove that (4n + 3)2 − (4n − 3)2 is
LHS = 30 +7=8 1 1 1 1 n divisible by 12 for all positive integers n.
g    ...   b n2 + 37n + 37 is a prime number.
So P(0) is true. 1 2 2  3 3  4 n(n  1) n  1
c 13 + 33 + 53 + … + (2n − 1)3 = n2(2n2 − 1)
Assume that the statement is true for some Assumption. h n3 − n is a multiple of 6 or all n ∈ +.
n(n2  1)

F
k ∈ , k ≥ 0. d 1  2  2  3  3  4  ...  (n  1)  n 
i 2n+2 + 32n+1,   n ∈ + is divisible by 7.
3
ie 32k + 7 = 8A n(2n  1)(2n  1)
j 12  32  52  ...  (2n  1)2  e 
n3 − n is divisible by 3 for all values of
⇒ 32k = 8A − 7,   A ∈ + 3 n ∈ +.
n
n
When n = k + 1: Use the assumption. k ∑ r(r + 1) = 3 ( n + 1) ( n + 2)
r =1

A
LHS = 32(k+1) + 7 n
1 n
9  3  72k l ∑ r(r + 1) = n + 1
r =1

 9(8 A  7)  7
 72A  63  7
 72A  56

R
 8(9 A  7) (9A − 7) ∈ +, since A ∈ +.
 8B 1.4 Counting principles and the
Since P(0) was shown to be true and it
was also shown that if the statement is
binomial theorem
true for some n = k, k ∈ +, it is also true

D
for n = k + 1, it follows by the principle of
mathematical induction that the statement Investigation 15
is true for all positive integers. Mary creates a fun game for practising some mathematics. She arranges 10
cups, numbers them as shown in the diagram on the right, and places one 1
marble just outside cup number 1. She then writes the following instructions.
Exercise 1G 2 3
Instructions for play:
• The number of marbles you place in each cup is equal to the number of 4 5 6
1 
Use the diagrams to answer the questions b If the diagrams were to continue what the cup multiplied by the number of marbles in the previous cup.
below. would the next two series be?
• The starting point is cup 1, where you will multiply the number on the 7 8 9 10
c Write a conjecture based on your cup by the number of marbles outside of cup 1.
findings.
1 If you were to follow these instructions, find how many marbles would be placed in:
d Prove your conjecture using a direct proof. a cup number 2 b cup number 3
e Prove your conjecture using the c cup number 5 d cup number 8.
principle of mathematical induction.
2 If Mary places another two rows underneath this arrangement, how many cups would there be in total?
2 Use mathematical induction to prove the
following statements: 3 How can you represent the number of marbles that would be placed in the last cup?
a 
Each of the three diagrams represents a
1 1 4 Comment on your results.
series. a 12  22  32  ...  n
2
n(n  1)(n  )
3 2
i Write down a series based on the line 5 How would you represent the number of marbles in the last cup if there were n cups in total?
divisions. n 1 n 1 n(n  1)
b 1  4  9  16  ...  (1) n  (1)
2

ii Write down a series based on colour. 2

50 51
1 FROM PATTERNS TO GENERALIZATIONS: SEQUENCES, SERIES AND PROOF 1.4
Factorial notation
Example 44

Number and algebra


The numbers in Investigation 15 got large very quickly. We can denote
these numbers mathematically as follows: Simplify the following.
The first marble outside the cup = u0 = 1. n  n  1 ! n! − (n − 1)!
a b
For the other cups, un = n × un−1. n! (n + 1)!

T
If you were to build up each term of this sequence, you would end up n  n  1 ! n  n  1  n! Rewrite (n + 1)! as (n + 1) × n!
a   n  n  1
with the result un = n × (n − 1) × (n − 2) × … × 3 × 2 × 1. n! n!
Some mathematical problems about arrangements and combinations
n!  (n  1)! n(n  1)!  (n  1)! Rewrite n! as n(n − 1)! and (n + 1)! as
involve counting techniques that use this sequence. A simpler way to b 
(n + 1) × n × (n − 1)!
denote the sequence is to use factorial notation where un = n!, (n  1)! (n  1)  n  (n  1)!

F
u0 = 0! = 1. (n  1)!  n  1  n  1
 
Here are the first five factorial numbers: (n  1)  n  (n  1)! n  n  1
0! = 1
1! = 1 × 0! = 1

A
2! = 2 × 1! = 2
Permutations and combinations
3! = 3 × 2! = 6
4! = 4 × 3! = 24
Investigation 16a
Angela is creating invitation cards for her birthday party. She has three images

R
n! = n × (n − 1)! = n × (n − 1) × (n − 2) ×   …   × 3 × 2 × 1
which she wants to use on her invitation. She puts the images in a row as
shown below.
This pattern lends itself to calculating expressions with very large
numbers, as shown in the following examples.

D
Example 43
Find the value of these expressions:
8!  4! 7! × 5!
a 7! b 3! c d
5! 5! 10! 10! × 6!

You can also find these using technology as She would like to consider all the possible ways of arranging these images in
7! 7  6  5  4  3  2  1
a   42 follows: one line on a card and then choose her favourite.
5! 5 4 3 21
1 How many arrangements can she choose from?
3! 3 21 1 2 How did you come up with your answer?
b  
5! 5  4  3  2  1 20
3 Angela realizes that she should also include the address, and she still
 4! 4  3 21 wants to leave the four objects in a line. In how many ways can this be
c 8! 4!
 
4
done?
10! 10  9 10  9 15
4 She then decides that she does not need to include her address on
d 7!  5!
 
1 1 invitations to her cousins, and realizes that if she makes all invitations
10!  6! 10  9  8  6 4320 individual she will have just enough different invitation cards for all her
guests. How many people is she going to invite to her party?
5 What happens if she wants to invite another friend to the party? Explain
your answer.

52 53
1 FROM PATTERNS TO GENERALIZATIONS: SEQUENCES, SERIES AND PROOF 1.4
Most probably, when responding to Investigation 16a, all the different
Investigation 16c

Number and algebra


arrangements (called permutations) were listed. Here is another way
of reasoning out the responses: Let’s consider what happens if we want to choose three C ABC
Suppose you want to find the total number of arrangements of the letters out of five and represent these on a chart similar B D ABD
letters A, B and C. You have three letters to choose from for the first letter. to the one discussed for permutations. If the first letter
E ABE
chosen is A then we have the chart shown here.

T
Having chosen this first letter, you have two B C ABC 1 What do you notice about the colour coded B ACB
choices for the second letter, and then you arrangements on the right? C D ACD
A
are left with only one letter to complete the
C B ACB E ACE
whole set. In other words, Angela has 3 × 2 × 2 What would you expect to notice if the first letter A
1 ways of designing the invitation cards for her chosen had been B? B ADB

F
cousins. You can think of this method as filling 3 What if you were to consider all the possible D C ADC
A C BAC
boxes as shown here, starting from left to right. permutations? E ADE
B
This reasoning can be extended to deduce that
the number of ways in which n distinct objects
4 If the order of choosing the letters is not B AEB
C A BCA
can be arranged in a row is
important, how can you derive the number of E C AEC
combinations? D AED
n × (n − 1) × (n − 2) × … × 3 × 2 × 1 = n!

A
A B CAB

The number of ways of arranging n distinct HINT


C When the order of arrangements is not relevant we speak about
objects in a row is n! combinations. In some books you may find
B A CBA
 n
the notation   for nCr .
r

R
Now suppose that you have five different letters and you want to The number of ways of choosing (when order is not important)  
find the number of possible arrangements of just three of these n! In this book we will use the
letters. You can choose the first letter in five ways, the second r objects from n distinct objects is nC r  latter which is the notation
r !(n  r)!
letter in four ways and the third letter in three ways, giving: you will encounter in IB
5  4  3  2 1 5! TOK exams.

5 4 3  ways.
2 1 (5  3)!
How many different

D
Using the same reasoning, you can deduce that the number of ways
of arranging three objects chosen out of n distinct objects would be
tickets are possible in a Example 45
n! lottery?
n   n  1   n  2  . And generalizing even further: a In how many ways can the letters of the word candle be arranged?
 n  3 ! What does this tell us
b In how many ways can a group of four boys and three girls be arranged?
about the ethics of selling
lottery tickets to those c In how many ways can a group of four boys and three girls be arranged if no girls are to
The number of permutations of r objects out of n distinct objects is given by who do not understand be next to each other?
n! the implications of these
n
Pr = . a There are six letters in the word candle, Using a calculator:
(n − r)! large numbers?
which can be arranged in 6! = 720 ways.
b Seven children can be arranged in
Investigation 16b 7! = 5040 ways.
c 4! ways of arranging the boys.
Let’s revisit Investigation 16a. Angela now wants to choose a photo of the
pool for her third image. She has five pool photos and decides to choose two 3! ways of arranging the girls.
different images, one for the invitation to relatives and another one for the If the girls are to be separated, then each
invitation to friends. pair of girls must be separated by at least
1 In how many ways can she choose the first photo? one boy.
2 In how many ways can she choose the second photo? There are five positions each girl can fill,
at each end or the slots between boys,
3 It is irrelevant which photo to use on the two sets of invitations. In how many
as shown in the diagram, giving the
ways can she choose two photos out of five?
number of combinations 5C3.
4 Why is this answer different to arranging two photos chosen out of five?
Continued on next page

54 55
1 FROM PATTERNS TO GENERALIZATIONS: SEQUENCES, SERIES AND PROOF 1.4

Number and algebra


The total number of arrangements is From the total number of ways of We cannot have a team with no boys as
therefore 4! × 3! × 5C3 = 1440. choosing the team (answer a) we need there are not enough girls to form a team.
to exclude these three combinations, ie
B B B B
Using technology:
13C6 − 8C55C1 − 8C65C0 − 8C15C5 = 1400
Or

T
There are three ways to consider, two
So total number of arrangements is boys and four girls or three boys and
4! × 3! × 10 = 1440 three girls or four boys and two girls,
giving
8C2 × 5C4 + 8C3 × 5C3 + 8C2 × 5C4 = 1400

F
Example 46
How many four-digit numbers can be made using each of the following digits only once?
a 5, 6, 7 and 8 b 5, 6, 7, 8 and 9 c 7, 8, 9 and 0 Exercise 1H

A
a 4P4 = 4! = 24 There are 4! ways of creating a four-digit 1 
Copy and complete the table below, a In how many different ways can the
number with these digits using each digit simplifying expressions as shown in the first books be arranged on the shelf?
5!
b
5P4  120 only once. row. b In how many ways can the books be
(5  4)!
arranged so that books of the same
8!    −   6! 6! (56 − 1) = 39 600
c 3 × 3P3 = 3 × 3! = 18 The number cannot start with 0. subject are grouped together?

R
9!    +   8!
So, there are three ways of choosing the 8 A safe has two dials, one with 26 letters and
first digit, and the next three digits can be 7!    −   6! one with the digits 0 to 9.
chosen from the three remaining digits 6!    +   5! In order to open the safe, Rose has to
which include 0. choose a code consisting of three distinct
(n + 1)!    −   n!
letters followed by two distinct digits.
n!    −   (n − 1)!

D
Determine how many different safe codes
n!    +   (n − 1)! are possible.
Example 47 (n + 1)!    +   n! 9 A delegation of five students is to be
There are eight boys and five girls who attend the Senior Mathematics Club. Find how many selected for a Model United Nations
2 Find the value of: conference. There are 10 boys and 13 girls
ways the teacher can choose a team of six students to represent the school in a competition if:
a 8! b 4!  5! c 10!  8! to choose from.
a There are no gender restrictions. 4  6! 3!  6! 11!  6! a In how many different ways can a
b The team is to be made up of three girls and three boys. 3 Simplify the following: delegation be chosen if there are no
c At least two of each gender are included in the team. (n  1)! n!   n  1! (n!)2  1
restrictions?
a b c b If the team is to include at least one girl
13! n!  (n  1)! n! n!  1
a 13C6
  1716 We have to use combinations as the order and one boy, in how many ways can the
6!(13  6)!
4 Show that (2n  2)!(2n!)  2(2n  1).
2
of choosing is not important. delegation be selected?
b 8C3 × 5C3 = 560 We now need to choose three boys out of  n  1! (2n)! n 1
10 a 
How many four-digit numbers can be
eight and three girls out of five. 5 Solve for n ∈ +,   nC2 = 66. made using the digits 0, 1, 3, 4, 5, 8 and 9?
c We cannot have a team with only one
6 Solve the equation b How many of the four-digit numbers
girl or with no girls.
16(n − 1)! = 5n!    +   (n + 1)! where n ∈ +. have no repeated digits?
We also cannot have a team with only
c How many four-digit even numbers can
one boy. 7 On a bookshelf there are four mathematics
be made using the digits?
books, three science books, two geography
books and four history books. The books d How many of these even numbers are
are all different. divisible by 5?

56 57
1 FROM PATTERNS TO GENERALIZATIONS: SEQUENCES, SERIES AND PROOF 1.4
11 Graeme is training for a 10 km run. He has six 12 A group of 12 people want to go to a
The binomial theorem states that for all n ∈ +, a, x ∈ .

Number and algebra


different routes to choose for his training and concert. They can travel in a small car that
he trains four times a week. He calculates that takes one driver and one passenger and  a  x  nC 0a n  nC1a n1 x  nC 2a n 2 x 2  ...  nCr a nr x r  ...  nC n x n
n

he will just manage to run a different set of two cars each taking one driver and four n
routes each week leading up to his next race. passengers. If there are five drivers in the   nC r a n r x r
How many weeks are there before Graeme’s group, in how many different ways can r 0

they travel?

T
race?

Investigation 19 TOK
The binomial theorem
1 Write the expansion of (1 + x)n using combinations. Why do we call this

F
Pascal’s triangle when it
Investigation 17 2 Find how many terms this expansion has when: was in use before Pascal
a n = 4, 6, 10 b n is even was born?
Copy and complete this table by using repeated algebraic multiplication. c n = 3, 5, 7 d n is odd.
Are mathematical
3 The pattern found in Investigation 17 is known as Pascal's triangle. The
(1 + x)i Constant Coefficient of x Coefficient of x2 Coefficient of x3 Coefficient of x4 Coefficient of x5 theories merely the
following properties of this pattern were found:
collective opinions of

A
(1 + x)0 1 - - - - -
• There is a line of symmetry going down the middle of the numbers. different mathematicians,
(1 + x)1 1 1 - - - - • Each row starts and ends with 1 and each of the other numbers is the or do such theories give
(1 + x)2 sum of the two numbers above it to either side. us genuine knowledge of
(1 + x)3 The expansion of (1 + x)5 can be written as follows: the real world?
(1 + x)4 (1 + x)5 = 5C0x0 + 5C1x + 5C2x2 +  5C3x3 + 5C4x 4  + 5C5x5                 

R
(1 + x)5 a Use this expansion and the one for (1 + x)6 to verify the two
properties above.
1 Comment on any patterns that you recognize. b Write the expansions for (1 + x)n and (1 + x)n+1.
2 Rearrange the numbers so that rather than forming a right-angled triangle, they form an isosceles c Use the same method as in part a to show that the two properties
triangle with 1 at the top vertex. above hold.
d  Conceptual How can you explain the patterns in Pascal’s triangle by

D
3 What new patterns do you notice now? considering the general expansion of the binomial expansion?
4 If you extended this pattern, what would you get in the next row?
5 Verify your response to question 4 using algebraic multiplication.
Example 48
Investigation 18 Find the values of a, b and c in the following identities:
Consider the expansion (1 + x)3 = (1 + x)(1 + x)(1 + x). a (1 − 2x)5 ≡ 1 + ax + bx2 + cx3 +   …   − 32x5
a 8
1 How is the constant term obtained in this expansion?  x x
b 1    1  bx  cx 2  ....   
 3   3
2 How is the term in x obtained?
c (2 + ax)7 ≡ b + 224x + cx2 + 70x3 + …
3 What about the term in x2?
a (1 − 2x)5 ≡ 5C0 + 5C1(−2x) + 5C2(−2x)2 + 5C3(−2x)3 You should know the first five
4 And the term in x3? +   …   + 5C5(−2x)5 rows of Pascal’s triangle.
5 Summarize your responses using mathematical notation. = 1 + 5 × (−2x) + 10 × (−2x)2 + 10 × (−2x)3 + ... + 1 × (−2x)5 (1 − 2x)5 = (1 + (−2x))5
6  Conceptual Repeat the process for the expansion = 1 − 10 x + 40 x − 80 x + ... − 32x
2 3 5

(a + x)3 = (a + x)(a + x)(a + x) ⇒ a = −10, b = 40 and c = −80


7 Write a conjecture for obtaining the expansion of
a  x  
a  x  a  x  a  x  ...  a  x  Continued on next page
n

 
n factors

58 59
1 FROM PATTERNS TO GENERALIZATIONS: SEQUENCES, SERIES AND PROOF 1.4
Example 51

Number and algebra


a 8
 x x  1 
6
b 1    1  bx  cx 2  ....    Find the term independent of x in the expansion of  x 2  .
 3 3  2x 
⇒a=8
6
 x
8
x 8 x
2
x
8
 1 
The general term in the expansion of  x 2  Write the general term of the
1 + 3  ≡ 1 + C1  3  + C 2  3  + ... +  3  x 
8

T
        is given by:  2 expansion.
8 r r
8! x 8! x2 x  1   1  12 2r r
 
6 r
=1 + × + × + ... +   6
Cr x 2   2x   C r   2  x
6
1!7! 3 2!6! 9 3    
8
8 28 2 x For the term independent of x: For the term independent of x, the
=1+x + x + ... +  

F
3 9 3 12 − 3r = 0 ⇒ r = 4
total power of x must be 0.
8 28
⇒ b = and c =  1 
4
 1
4
6! 1 15
 
6 4
3 9   2x   C4    x 0   
6
C4 x 2 6
Give your answer as an exact fraction.
   2 4!2! 16 16
c (2 + ax)7 ≡ 27 + 7C1 × 26 × (ax) + 7C2 × 25 × (ax)2
 + 7C3 × 24 × (ax)3 + …

A
7! 7! 7!
 128   26 ax   25 a 2 x 2   24 a3 x 3  ... Exercise 1I
1!6! 2!5! 3!4!
128  448ax  672a 2 x 2  560a3 x 3  ... 1 Write the first four terms in the binomial 7 Write in factorial notation:
 448a  224 expansion of: a the coefficient of x4 in the expansion of

R
1  x
11
 x 
7
2
8
(1 + x)n+1

a ,
b 128 and 
c 168 a 1   b 1   c  x  
2 3  2    x b the coefficient of x3 in the expansion of
2 In each of the following binomial (1 + 2x)n.
expressions, write down the required term. c Find n, given that these two coefficients
a fifth term of (a − 2b)10 are equal.
Example 49 8 a Express  3 2 
5

D
11
 4 in the form of
Find the coefficient of x3y3 in the expansion of (x + 3y)6 .
b third term of  a  2 
 a  a 3  b 2 where a, b ∈ .
8 4
The general term in the expansion of (x + 3y)6 is given by 6Crx6−r(3y)r.  2y   1 
c fourth term of  x   b Express  2   in the form a  b 10 ,
We want 6Crx6−r(3y)r = Ax3y3 ⇒ r = 3.  x   5
6! 3 Find the term independent of x in the a, b ∈ .
Then A  6C3  33   27  540.
   
12 7 7
3!3!  2 c Express 1  5  1  5 in the form
expansion of  x  2  .
 x 
4 Use the binomial theorem to expand a 5 , a ∈ .
4
 x 9 Find the value of the following by choosing
 2  . Hence find the value of (1.99)4
Example 50  5  an appropriate value for x in the expansion
correct to 5 decimal places. of (1 + x)n.
Use the binomial theorem to expand (2x + 3y)5. Hence find the value of 2.035 correct to
5 decimal places. 5 Find the term in x6 in the expansion of a 
nC − 2  ×  nC + 4   ×  nC − 8   ×  nC +  … 
0 1 2 3
6 +  ( − 1)r 2r × nCr +  …  +  ( − 1)n 2n × nCn
 2 1
(2x + 3y)5 = (2x)5 + 5(2x)4(3y) + 10(2x)3(3y)2 + 10(2x)2(3y)3 + 5(2x)(3y)4 + (3y)5 x  x  . b 
nC +   nC1 +  nC2 + nC3 +  …nCr +  …   + nCn
  0
5
= 32x5 + 240x4y + 720x3y2 + 1080x2y3 + 810xy4 + 243y5  y
6 a Expand  x   .
When x = 1, y = 0.01 we obtain  x
( 2.03) b Find the coefficient of x3y2 in the
5
= 32 + 2.40 + 0.0720 + 0.001 080 + 0.00000810 + 0.0000000243
5
 y
 32.47309 (to 5 decimal places) expansion of  2x  y   x   .
 x

60 61
1 FROM PATTERNS TO GENERALIZATIONS: SEQUENCES, SERIES AND PROOF 1.4
Generalization of the binomial expansion TOK p

Number and algebra


It was around 1665 that Isaac Newton generalized the binomial The binomial expansion for (1 + x)α for    and −1 < x < 1 is given by the infinite
Is it possible to know q
theorem to allow for negative and fractional exponents. Let’s try to series:
examine this using some facts which were established earlier in this things about which we
can have no experience, α(α − 1) 2 α(α − 1)(α − 2) 3 α(α − 1)(α − 2)...(α − r + 1) r
(1 + x )
α
chapter. = 1+ α x + x + x + ... + x + ...
such as infinity? 2! 3! r!
Consider the geometric series 1 + x + x2 + x3 + …, where x is not equal to 0.

T
For which values of x does this series converge?
What is the sum to infinity for this series when it converges? Investigation 20
We can write the answers to these two questions as follows: 1 Factual Show that the generalized form for negative integer powers given above can be written as

F
1 
For −1 < x < 1, S 
1 x  nr 1Cr x r
(1  x) n 
r 0
In other words:
The table below includes the results obtained above.
1 International-
(1  x)1 1  x  x 2  x 3  ... Coefficient Coefficient Coefficient Coefficient Coefficient
1 x mindedness (1 − x)i Constant of x of x2 of x3 of x4 of x5

A
If we want to expand (1 − x)−2 we could say that this is equivalent to We cannot simply (1 −  x)−1 1 1 1 1 1 1…
((1 − x)−1)2 = (1 − x)−1(1 − x)−1 take it for granted
(1 −  x)−2 1 1 3 4 5 6…
= (1 + x + x2 + x3 + …)(1 + x + x2 + x3 + …) that we can multiply
out two infinite (1 −  x)−3 1 -3 +6 -10…
 1  x  x 2  x 3  x 4  ... multiplying the terms in the left bracket by 1 series easily. Gustav (1 −  x)−4
 x  x 2  x 3  x 4  ... multiplying the terms in the left bracket by x Dirichlet, a German

R
(1 −  x)−5
 x  x  x  ...
2 3 4
multiplying the terms in the left bracket by x 2
mathematician,
proved how this can 2 Use Newton’s generalization to verify that the coefficients shown in the table are correct.
...
be done. 3 Apply Newton’s generalization to copy and complete the table.
1  2x  3x 2  4 x 3  ...
4  Factual Show that the generalized form of the binomial theorem for fractional powers can be written as
⇒ (1 − x)−2 = 1 + 2x + 3x2 + 4x3 + … 

D
Similarly, we can repeat the process to obtain the expansion of (1 − x)−3:   Cr x r
(1  x) 
r 0
−3
(1 − x) = (1 − x) (1 − x) −2
5 Use the generalization to find the expansion of 1  x  and 1  x  .
( )(
= 1 + 2x + 3x 2 + 4 x 3 + ... 1 + x + x 2 + x 3 + ... )
= 1 + 2x + 3x + 4 x + ... 2 3

+ x + 2x 2 + 3x 3 + ...
Example 52
+ x 2 + 2x 3 + ... Expand the following up to the term in x3.
1 2 , for x  1
... a (1  2x) , for x  b
2 (1  3x) 3
= 1 + 3x + 6 x 2 + 10 x 3 + ....
1

Newton generalized this result for negative and rational exponents of a (1  2x) 1  2x  2 Using
(  1) 2
1  x  

the binomial theorem as follows:  1   1   2x   1  1  3   2x  1 x 
2 3
1 x
1   2x                ... 2!
2  2   2  2!  2  2  2  3! (  1)(  2) 3
1 2 1 3  x  ...
The binomial expansion for (1 − x)−n for   n ∈ + and −1 < x  < 1 is given by the infinite series: 1  x  x  x 3!
2 2 Simplify.
n(n + 1) 2 n(n + 1)(n + 2) 3 n(n + 1)(n + 2)...((n + r − 1) r
(1 − x )
−n
= 1 + nx + x + x + ... + x + ... Continued on next page
2! 3! r!

62 63
1 FROM PATTERNS TO GENERALIZATIONS: SEQUENCES, SERIES AND PROOF 1.4
TOK

Number and algebra


2 Using
b  2(1  3x)1 "Mathematics may be defined as the economy of counting. There is no problem in the
(1  3x) n(n  1) 2
1  x 
n
1  nx  x whole of mathematics which cannot be solved by direct counting."
2!
 (1)(2) (1)(2)(3) 
2  1  (1)(3x)  (3x)2  (3x)3  ...  n(n  1)(n  2) 3 -E. Mach
 2! 3!   x  ...
3! To what extent do you agree with this quote?
 

T
 2 1  3x  9 x 2  27 x 3  ... 1
n ∈ + and x 
2  6 x  18 x 2  54 x 3  ... 3
Exercise 1J
x

F
1 
Expand the following up to the term in x3, 4 Show that  x  2 x 2  3x 3  4 x 4 .
1 (1  x)2
Example 53 given that x  .
2
5 Find the binomial expansion of
1+ x 1 2
Use the binomial expansion to show that 1 + x + x2, x < 1. 1 1 (2  3x)3 , x  .
1− x 2 a b 3
(1  x) (1  2x)2 6 a Find the first four terms of the binomial
Using 1

A
1 x 1 1
2 2 expansion of 1  4 x , x  .
1 x  2 1  x  2

 (  1) c d 4
1  x 

1 x 1 x 
 x2 (1  2x) (1  x)3
2! b Show that the exact value of 1  4x
1
1  1  1  x
2
(  1)(  2) 3 2 Find the first four terms of each of the 1
1  x  2 1    x        ...  x  ... following expansions where x 
1
: when x  is
2 6
.
 2  2   2  2! 3! 100 5
10
x x 2
1 3 c Hence, determine 6 to 5 decimal
1    ... where  

R
2 8 2 a (1  2x) b (1  x)2 places.
We only need the expansion until 1 1
the term in x2. c 1  3x  2

d 2(1  x)3 7 a Find the first three terms of the binomial
1 1
Using expansion of where x  .
1 x x x 2 3
1  2x 2
(  1) 2 3 
Show that  1 x   ,
1  x  

1 x  x 1 x 2 2 b Hence or otherwise, obtain the
2!

D
where x  1. (2  3x)3 1
(  1)(  2) 3 expansion of , x  .
 x  ... 1  2x 2
3!
1
 1   3  x 
2
1
 1 where    , and noting that we
1  x   1     x        

2  ... 2
 2  2   2  2! have −x inside the brackets.
x 3x 2
1  
2 8
 ... We only need the expansion until
the term in x2.
Chapter summary
1 1  x x2  x 3x 2 
• Sequences may be finite or infinite.
(1 + x ) 2 (1 − x )

2  1 + − + ...  1 + + + ...  Multiply the two expansions up to • If the difference between two consecutive numbers in a sequence
 2 8  2 8  and including terms in x2. is constant then it is an arithmetic sequence or an arithmetic
 x 3x 2 progression. The constant difference is called the common
1 + +
difference and is denoted by d.
 2 8
1 1
 x x2 x2 • An arithmetic sequence with first term u1 and common difference d has
(1 + x ) 2 (1 − x )

2   + + + ...  1 + x + general term un = u1 + (n − 1)d.
 2 4 2
 x 2 • The sum of a finite arithmetic series is given by
 − n n
 8 Sn 2u1  (n  1)d  u1  un  where n is the number of terms in
2 2
the series, u1 is the first term, d is the common difference and un is the
last term.
Continued on next page

64 65
1 FROM PATTERNS TO GENERALIZATIONS: SEQUENCES, SERIES AND PROOF 1
• •

Number and algebra


If the ratio of two consecutive terms in a sequence is constant then it is The binomial expansion for (1 − x)−n for   n ∈ + and −1 < x  < 1 is
a geometric sequence or a geometric progression. We call the constant given by the infinite series:
ratio the common ratio and denote it by r.
n(n  1) 2 n(n  1)(n  2) 3
1  x 
n
• A geometric sequence with first term u1 and common ratio r has general 1  nx  x  x  ...
2! 3!
term un = u1r n−1, r ≠ 1, 0, −1, u1 ≠ 0. n(n  1)(n  2)...(n  r  1) r

T
 x  ...
• The sum of a finite geometric series is given by r!
n
u1(1  r ) p
Sn , r 1 • The binomial expansion for (1 + x)α for  
1 r q
where n is the number of terms, u1 is the first term and r is the and −1 < x < 1 is given by the infinite series:

F
common ratio.
(  1) (  1)(  2)
1  x 
n 
u1(1  r ) 1 x 
 x2  x 3  ...
• The sum of n terms of a geometric
series is Sn
1 r
, r  1. 2! 3!
When −1 < r < 1, rn approaches zero for very large values of n. The (  1)(  2)...(  r  1) r
 x  ...
u1 r!
series therefore converges to a finite sum given by S  .

A
1 r
• A proof in mathematics often consists of a logical set of steps that
validates the truth of a general statement beyond any doubt.
• A direct proof is a way of showing the truth of a given statement by
constru­cting a series of reasoned connected established facts. In a
direct proof the following steps are used:
Developing inquiry skills

R
m Identify the given statement. <To come>
m Use axioms, theorems, etc, to make deductions that prove the
conclusion of your statement to be true.
• When setting out a proof by contradiction you follow the following
steps:

D
m Identify what is being implied by the statement.
m Assume that the implication is false.
m Use axioms, theorems, etc … to arrive at a contradiction.
m This proves that the original statement must be true.
• A counterexample, or counterclaim, is an acceptable “proof” of the fact
that a given statement is false.
• n! = n × (n − 1)! = n × (n − 1) × (n − 2) ×   …   × 3 × 2 × 1 Chapter review Click here for mixed
review exercise
• The number of ways of arranging n distinct objects in a row is n!
• The number of permutations of r objects out of n distinct objects is given 1 
Show that there are two geometric 4 a P
 rove the identity
n! sequences such that the second term is 9 1 1 x2 .
by n Pr = . and the sum of the first three terms is 91.   2
(n − r)! 1 x  2  2x  5x  3
Write the fourth term of each sequence. 3 1  x 
• The number of ways of choosing (when order is not important) r objects  3 
n! 2 
Find the sum of the series 1 + 2 + 3 + 4 + 5 + b  Hence, use the binomial expansion to
from n distinct objects is nC r  7 + 8 + 9 + 11 + 13 + 15 + 16 + 17 + … + 64. find the first four terms of the expansion
r !(n  r)!
x2
3 Three numbers a,  b  and c form an arithmetic of .
2x  5x  3
2
sequence. The numbers b,  c  and a form a
geometric sequence. Find the three numbers 5 Prove the following identities:
given that they add up to 36. a n+1C2 ≡ nC2 + n
b nC2  ×  n−2Ck−2 ≡ nCk × kC2

66 67
1 FROM PATTERNS TO GENERALIZATIONS: SEQUENCES, SERIES AND PROOF 1
6 
Show that nC0 + 3 × nC1 + 32 × nC2 + … + d Use the binomial theorem to write a4  rove by contradiction that for all n ∈  + ,
22 P 25 
Seven women and two men are chosen
× +…+ × =

Number and algebra


3r nC
r
3n nC
n
22n and b4 in terms of x and y and use your if n + 3 is odd, then n is even. (7 marks)
3 to sit in a row and have their photograph
result to factorize a4 − b4. taken.
7 Prove by contradiction that no two integers
a  and b can be found such that 14a + 7b = 1. e Use your results to make a conjecture 23 
Prove, by mathematical induction, that a How many different ways can they be
for the factors of an − bn. 52n−1 + 1 is divisible by 6 for all n ∈ . arranged? (1 mark)
8 Prove by contradiction that if x = 3 then f P  rove your conjecture using (8 marks)
5x − 7 ≠ 13. b 
How many ways can they be arranged

T
mathematical induction. 24 a Find the first four terms, in ascending if the men must sit together? (2 marks)
9 
Give a counterexample to prove that each 15 Given that the coefficients of xr−1,   xr   powers of x, of the binomial expansion
of the following statements is false: and xr+1 in the expansion of (1 + x)n c 
How many ways can they be arranged
of 3 (1 − x ) ( x < 1) . (4 marks) if the men must sit apart? (2 marks)
a If − < 0 then a − b > 0.
a2 b2 are in arithmetic sequence, show that
n2 + 4r2 − 2 − n(4r + 1) = 0. b Use your answer to part a) to find
b 3n + 2 is prime for all n ∈ +. d 

F
How many ways can they be arranged
Hence find three consecutive coefficients an approximation for 3 63 to six
c 2n  1 is irrational for all n ∈ +. decimal places. You must show all your
if there must be at least two women
of the expansion of (1 + x)14 which form separating the men? (3 marks)
d 2n − 1 is prime for all n ∈ +. an arithmetic sequence. working. (5 marks)
10 P
 rove by mathematical induction that 16 Given that
(1 × 1!) × (22 × 2!) × (33 × 3!) × (44 × 4!) × 2  x  7x 2 A B C
   ,

A
… × (nn × n!) = (n!)n+1 (1  2x)(1  x ) (1  2x) (1  x) (1  x)
2

11 U
 se mathematical induction to prove that determine the values of A, B and C.
n3 + 2n is a multiple of 3.
Hence use the binomial theorem to find the
12 
Use mathematical induction to prove the 2  x  7x 2
expansion of in ascending
following statements: (1  2x)(1  x 2 )

R
n
n(n + 1) n
n(n + 1)(2n + 1) powers of x up to and including the term in x3.
a ∑r = 2
b ∑r 2
=
6
Exam-style questions
r =1 r =1
n
n2(n + 1)2
c ∑r
r =1
3
=
4
Hence prove that 17 
Find the coefficient of the term in x5 in the
binomial expansion of (3 + x ) ( 4 − 2x ) .

D
8
n
n(n + 1)(n + 2)(n + 3)
r =1
∑ r (r + 1) (r + 2) = 4
(4 marks)
3 a I n how many ways can the letters of
1
the word harmonics be arranged?
18 
The coefficient of x2
in the binomial
expansion of (1 + 3x ) where n ∈ℚ is 495.
n

b  Determine how many numbers bigger


than 30 000, less than 9 999 999 and Determine the possible values of n.
divisible by 5 can be formed using the (6 marks)
digits 0, 1, 2, 3, 5, 8 and 9. n =15

c In how many ways can a committee 19 


Find the value of ∑ (1.6
n=0
n
)
− 12n + 1 , giving
of five people be selected from seven
men and four women, so that there is your answer correct to 1 decimal place.
at least one male and one female and (6 marks)
there are more women than men on the
committee? 20 Prove the binomial coefficient identity
14 L
 et a = x + y  and b = x − y.  n   n − 1  n − 1
 = +  . (6 marks)
a Write a2 − b2 in terms of x and y and  k   k   k − 1
hence show that a2 − b2 = (a − b)(a + b). 21 Find the sum of all integers between 500
b Use the binomial theorem to write a3 and 1400 (inclusive) that are not divisible
and b3 in terms of x and y. by 7. (7 marks)
c U  se your results to part b to show that
a3 − b3 = (a − b)(a2 + ab + b2).

68 69
1
The Towers of Hanoi Approaches to learning: Thinking skills,
Communicating, Research
Try a formula
Exploration criteria: Mathematical
communication (B) Personal engagement (C), Return to the problem with 4 disks.
The problem
Use of mathematics (E) Consider this image of a partial solution to the problem. The large disk
on peg A has not yet been moved.
IB topic: Sequences
Modelling and investigation activity

Modelling and investigation activity


T
1
2

F
3
4

A
A B C A B C
The aim of the Towers of Hanoi problem is to move all the disks from peg A to peg C following these rules:
Consider your previous answers.
1 Move only one disk at a time.
What is the minimum possible number of moves made so far?

R
2 A larger disk may not be placed on top of a smaller disk. How many moves would it then take to move the largest disk from peg
3 All disks, except the one being moved, must be on a peg. A to peg C?
When the large disk is on peg C, how many moves would it then take to
For 64 disks, what is the minimum number of moves needed to complete the problem? move the 3 smaller disks from peg B to peg C?
How many total moves are therefore needed to complete this puzzle?

D
Use your answers to these questions to write a formula for the minimum
Explore the problem number of moves needed to complete this puzzle with n disks.
Use an online simulation to explore the Towers of Hanoi problem for 3
This is an example of a recursive formula. What does that mean?
and 4 disks.
How can you check if your recursive formula works?
What is the minimum number of moves needed in each case?
What is the problem with a recursive formula?
Solving the problem for 64 disks would be very time consuming, so
you need to look for a rule for n disks that you can then apply to the Try another formula
problem with 64 disks.
You can also try to solve the problem by finding an explicit formula
that does not depend on you already knowing the previous minimum
Try and test a rule number.
Assume the minimum number of moves follows an arithmetic sequence.
You already know that the relationship is not arithmetic.
Use the minimum number of moves for 3 and 4 disks to predict the
How can you tell that the relationship is not
minimum number of moves for 5 disks.
geometric?
Check your prediction using the simulator. Extension
Look for a pattern for the minimum number of
Does the minimum number of moves follow an arithmetic sequence? moves in the table you constructed previously. • What would a solution look like for 4 pegs?
Hence write down a formula for the minimum Does the problem become harder or easier?
Find more results number of moves in terms of n. • Research the ‘Bicolor’ and ‘Magnetic’ versions
Use the simulator to write down the number of moves when n = 1 and n = 2. of the Towers of Hanoi puzzle.
How does an explicit formula differ from a recursive
Organize your results so far in a table. formula? • Can you find an explicit formula for other
Look for a pattern. If necessary, extend your table to more values of n. Use your explicit formula to solve the problem with recursive formulae? (e.g. Fibonacci)
64 disks.
70 71

Das könnte Ihnen auch gefallen